Решение уравнений примеры и решение: 100 примеров решения уравнений — просто и практично

Содержание

100 примеров решения уравнений — просто и практично

La решение уравнений делается через метод эквивалентных уравнений. Следует учитывать, что уравнения первой степени многочлены первой степени, равные нулю.

Примеры решения уравнений

Общая форма уравнения первой степени

ax + b = 0

где a y b его определенные коэффициенты в некотором числовом наборе и x вычисляемое значение, неизвестно.

Метод эквивалентных уравнений

Ключевым элементом каждого уравнения является символ равенства (=). Каждое уравнение представляет собой равенство между двумя терминами.

aх + b = cх + d

левый термин = правильный термин

Любое изменение, сделанное в одном члене уравнения, должно также быть сделано и в другом члене, чтобы это не изменило значение уравнения.

Таким образом, уравнения получены из по-другому, но сохраняют свое неизменное исходное значение.

Пример эквивалентных уравнений

  1. 6х + 2 = 4Икс — 9
  2. 6х + 1 = 4Икс — 10

Оба уравнения эквиваленты, бросай такое же значение х.

Разница между уравнением «а» и уравнением «б» в том, что второе уравнение такое же, как и первое, но из обеих частей вычитается 1.

Операция «Pasaв математике не существует.

 

Решенные примеры уравнений

уравнения с

целые коэффициенты

уравнения с

рациональные коэффициенты

Найти решение уравнений по метод эквивалентных уравнений это очень полезно при изучении химии и физики, при очистке уравнений, чтобы найти значение неизвестного.

Примеры уравнений с решениями

  1. – 13х – 6 = 0 → х = – 0,46153846
  2. – 15 + 6х = 14х + 26 → х = -22,8571429
  3. – 18х – 7 = 0 → х = – 0,38888889
  4. – 12 + 17 х = 12 х + 4 → х = 16 / 5
  5. 39 – 27 х = 1 х – 8 → х = 47 / 28
  6. – 23х – 8 = 0 → х = – 0,34782609
  7. 12 – 13 х = 8 х – 4 → х = 16 / 21
  8. – 28х – 9 = 0 → х = – 0,32142857
  9. – 20 + 8х = 13х + 27 → х = -30,0769231
  10. – 30х – 10 = 0 → х = – 0,33333333
  11. – 17 + 19 х = 11 х + 5 → х = 11 / 4
  12. – 32х – 11 = 0 → х = – 0,34375
  13. 17 – 15 х = 7 х – 5 → х = 1
  14. – 34х – 13 = 0 → х = – 0,38235294
  15. – 25 + 10х = 12х + 28 → х = -37,3333333
  16. – 36х – 15 = 0 → х = – 0,41666667
  17. – 22 + 21 х = 10 х + 6 → х = 28 / 11
  18. – 38х – 17 = 0 → х = – 0,44736842
  19. 22 – 17 х = 6 х – 6 → х = 28 / 23
  20. – 40х – 19 = 0 → х = – 0,475
  21. 45 – 33 х = 4 х – 5 → х = 50 / 37
  22. – 30 + 12х = 11х + 29 → х = -44,6363636
  23. – 42х – 21 = 0 → х = – 0,5
  24. – 32 + 14х = 10х + 30 → х = -49
  25. 10х – 3 = 0 → х = 3 / 10
  26. 27 – 19 х = 5 х – 7 → х = 34 / 24
  27. 15х – 4 = 0 → х = 4 / 15
  28. – 27 + 23 х = 9 х + 7 → х = 17 / 7
  29. 20х – 5 = 0 → х = 1 / 4
  30. 29 – 21 х = 4 х – 6 → х = 7 / 5
  31. 25х – 6 = 0 → х = 6 / 25
  32. – 34 + 16х = 9х + 31 → х = -53,4444444
  33. 27х – 7 = 0 → х = 7 / 27
  34. – 29 + 25 х = 8 х + 6 → х = 35 / 17
  35. 29х – 8 = 0 → х = 8 / 29
  36. 30 – 23 х = 3 х – 5 → х = 35 / 26
  37. 31х – 10 = 0 → х = 10 / 31
  38. – 36 + 18х = 8х + 33 → х = -58,125
  39. 33х – 12 = 0 → х = 12 / 33
  40. – 30 + 27 х = 7 х + 5 → х = 7 / 4
  41. 32 – 25 х = 2 х – 4 → х = 36 / 27
  42. 35х – 14 = 0 → х = 2 / 5
  43. – 38 + 20х = 7х + 35 → х = -63
  44. 37х – 16 = 0 → х = 16 / 37
  45. – 32 + 29 х = 6 х + 4 → х = 36 / 23
  46. 39х – 18 = 0 → х = 18 / 39
  47. – 10 = х – 3 → х = 10 / 3
  48. – 40 + 22х = 6х + 37 → х = -68,1666667
  49. – 15 = х – 4 → х = 15 / 4
  50. – 34 + 31 х = 5 х + 3 → х = 37 / 26
  51. – 20 = х – 5 → х = 4
  52. – 42 + 24х = 5х + 39 → х = -73,8
  53. – 25 = х – 6 → х = 25 / 6
  54. 34 – 27 х = -1 х – 3 → х = 37/26
  55. – 27 = х – 7 → х = 27 / 7
  56. 36 – 29 х = -2 х – 6 → х = 42/27
  57. – 29 = х – 8 → х = 29 / 8
  58. – 44 + 26х = 4х + 41 → х = -80,25
  59. – 31 = х – 10 → х = 31 / 10
  60. 41 – 29 х = 2 х – 7 → х = 48 / 31
  61. – 36 + 33 х = 4 х + 2 → х = 38 / 29
  62. – 33 = х – 12 → х = 33 / 12
  63. 38 – 31 х = -3 х – 5 → х = 43/28
  64. – 35 = х – 14 → х = 35 / 14
  65. – 38 + 35 х = 3 х + 1 → х = 39 / 32
  66. – 37 = х – 16 → х = 37 / 16
  67. 40 – 33 х = -4 х – 4 → х = 44/29
  68. – 39 = х – 18 → х = 39 / 18
  69. – 40 + 37 х = 2 х + 0 → х = 8 / 7
  70. 16 – 6 х = 12 х – 25 → х = 24,0833333
  71. 15 – 11 х = 11 х – 6 → х = 21 / 22
  72. 21 – 8 х = 11 х – 26 → х = 31,3636364
  73. 26 – 10 х = 10 х – 27 → х = 38,7
  74. 31 – 12 х = 9 х – 28 → х = 46,1111111
  75. 43 – 31 х = 3 х – 6 → х = 49 / 34
  76. 20 – 13 х = 10 х – 7 → х = 27 / 23
  77. 33 – 14 х = 8 х – 29 → х = 50,625
  78. 25 – 15 х = 9 х – 8 → х = 33 / 24
  79. 35 – 16 х = 7 х – 30 → х = 55,2857143
  80. 30 – 17 х = 8 х – 9 → х = 39 / 25
  81. 37 – 18 х = 6 х – 32 → х = 60,3333333
  82. 32 – 19 х = 7 х – 8 → х = 20 / 13
  83. 39 – 20 х = 5 х – 34 → х = 65,8
  84. 33 – 21 х = 6 х – 7 → х = 40 / 27
  85. 41 – 22 х = 4 х – 36 → х = 72
  86. 35 – 23 х = 5 х – 6 → х = 41 / 28
  87. 43 – 24 х = 3 х – 38 → х = 79,6666667
  88. 37 – 25 х = 2 х – 5 → х = 42 / 27
  89. 45 – 26 х = 2 х – 40 → х = 91

предлагаемая деятельность:

Выберите несколько результатов с повторяющимися десятичными выражениями и вместо вычисления всех этих десятичных знаков запишите результат в виде дроби.

« 15 примеров резки

10 примеров описательного текста »

Линейные уравнения

Сегодня мы познакомимся с линейными уравнениями. Узнаем, как их решать. Разберём и простые примеры, и довольно хитрые. Это один из важнейших уроков в курсе алгебры 7 класса.

Содержание

  1. Краткая вводная по уравнениям
  2. Что такое линейное уравнение
  3. Решение простых уравнений
  4. Более сложные задачи
  5. Практика: 3 дополнительных уравнения

1. Краткая вводная по уравнениям

Уравнение — это любое равенство, в котором присутствует хотя бы одна переменная.

Примеры равенств и уравнений.

  • Равенство $5-3=2$ — это не уравнение. Да, оно верное, но в нём нет переменной.
  • Равенство $5+3=2$ — тоже не уравнение. Оно ещё и само по себе неверное.
  • А вот равенство $5-x=2$ или $5+3x=2$ — это уравнения. В них есть переменная $x$.

Мы знаем, что равенства могут быть верными, а могут быть и неверными. Чтобы проверить это, достаточно вычислить выражение, стоящее с каждой стороны от знака «равно» и сравнить полученные значения: если числа слева и справа одинаковые, то равенство верно. А если числа получились разные — равенство неверное.

С уравнениями всё сложнее. Их нельзя просто взять и вычислить, потому что мы не знаем, какое значение принимает переменная. Но если вместо переменной подставить какое-либо число, то уравнение превращается в обычное равенство — и дальше всё легко.

Пример 1. Рассмотрим уравнение: $x+5=8$.

Если подставить $x=10$, получим равенство $10+5=8$, которое, очевидно, не верно.

Но если $x=3$, то получится $3+5=8$ — это верное равенство.

Итак, есть значения переменных, при которых уравнение обращается в верное числовое равенство. А есть значения, при которых равенство получается неверным. Это позволяет ввести понятие корня уравнения.

Определение. Корень уравнения — это такое значение переменной, при подстановке которого это уравнение обращается в верное числовое равенство.

Решить уравнение — значит найти все его корни, либо доказать, что таких корней нет.

Существует бесчисленное множество разных уравнений. Одни решаются легко, другие вообще не решаются.

Умение решать такие уравнения — это сложный и очень ценный навык. И сегодня мы начнём осваивать этот навык. Для этого рассмотрим самый простой вид уравнений — линейные.

2. Что такое линейное уравнение

Определение. Линейным уравнением называется уравнение вида $ax+b=0$, где $a$ и $b$ — числа, $x$ — переменная.

Также линейными называют все уравнения, которые сводятся к виду $ax+b=0$ путём элементарных преобразований. {2}} &=0 \\ \frac{5}{x} &=1 \\ \left| x \right| &=64 \end{align}\]

Ещё раз: линейные уравнения могут выглядеть очень по-разному. Но все они сводятся к виду $ax+b=0$ с помощью элементарных преобразований. По таким преобразованиям у нас будет отдельный урок, а сейчас просто вспомним, что это такое.

2.1. Элементарные преобразования уравнений

Существует ровно три вида преобразований, которые называются элементарными:

  • 1.Прибавить к обеим частям уравнения одно и то же выражение.
  • 2.Умножить обе части уравнения на одно и то же выражение, отличное от нуля.
  • 3.Поменять местами выражения, стоящие слева и справа от знака равенства.

Замечательное свойство всех этих преобразований состоит в том, что они не меняют корни уравнения. Но при этом зачастую позволяют получить уравнение, разрешённое относительно переменной, т.е. уравнение вида $x=a$, где $a$ — некоторое числовое выражение, которое уже не содержит переменную $x$.

Пример 3. Решите уравнение: $x+5=18$.

Вычтем из обеих частей пятёрку:

\[\begin{align}x+5-5 &=18-5 \\ x &=13 \end{align}\]

Получили $x=13$ — это и есть корень.

Иногда переход от уравнения $x+5=18$ к уравнению $x=18-5$ называют «переносом слагаемого их левой части в правую». Мы тоже будем так говорить. Но помните: во «взрослой» алгебре (а именно такой мы будем заниматься с 7 по 11 класс) никаких «переносов» нет. Есть только прибавление слагаемых (пускай и противоположных к исходным).

3. Решение простых уравнений

Итак, у нас есть уравнение $ax+b=0$. Первое, что хочется сделать — это перенести слагаемое $b$ вправо, а затем разделить всё на $a$:

\[\begin{align}ax+b &=0 \\ ax &=-b \\ x &=-\frac{b}{a} \end{align}\]

С первым шагом проблем возникнуть не должно: мы вправе прибавлять к обеим частям уравнения любое выражение, в т.ч. $-b$:

\[\begin{align}ax+b-b &=0-b \\ ax &=-b\end{align}\]

А вот дальше начинаются проблемы. Если коэффициент $a\ne 0$, то снова никаких проблем: мы вправе поделить обе части уравнения на любое ненулевое выражение, в т.ч. на это самое $a\ne 0$:

\[\begin{align}ax &=-b \\ \frac{ax}{a} &=-\frac{b}{a} \\ x &=-\frac{b}{a} \end{align}\]

Большинство уравнений действительно так и решаются. Взгляните на примеры:

Пример 4. Решите уравнение: $5x=10$.

Просто делим обе части уравнения на 5:

\[\begin{align}5x &=10 \\ x &=2 \end{align}\]

Получили $x=2$ — это и есть искомый корень.

Пример 5. Решите уравнение: $-8x=48$.

Всё то же самое, просто делим на отрицательное число:

\[\begin{align}\frac{-8x}{-8} &=\frac{48}{-8} \\ x &=-6 \end{align}\]

Корень уравнения: $x=-6$. То, что он отрицательный, нисколько не должно нас смущать.

Но что делать вот с такими уравнениями?

\[0\cdot x=10;\quad 0\cdot x=0\]

В первом случае корней вообще нет. Потому что при любом значении $x$ мы умножаем это значение на ноль и получаем ноль, который никак не может равняться 10.

Во втором уравнении корнем наоборот будут все числа. Потому что опять же любое число при умножении на ноль даст ноль — и именно этот ноль от нас и требуется.

3.1. Основной алгоритм

Итого мы получаем три варианта развития событий. Пусть дано уравнение $ax+b=0$. Тогда:

  • 1.Если $a\ne 0$, то уравнение имеет один корень: $x=-{b}/{a}\;$.
  • 2.Если $a=0$, но $b\ne 0$, то корней нет.
  • 3.Если же $a=0$ и $b=0$, то корни — все числа.

Вот так всё просто. Однако я не хочу, чтобы вы просто зазубрили эти три пункта и бездумно применяли их, когда видите линейное уравнение. Пожалуйста, помните, как и почему возникают эти правила, что такое элементарные преобразования и какие ограничения в них присутствуют (на самом деле ограничение лишь одно: нельзя умножать и делить на ноль).

Пример 6. Решите уравнение: $7x-2=6+3x$.

Вычитаем из обеих частей $3x$ и добавляем 2:

\[\begin{align}7x-2 &=6+3x|-3x+2 \\ 4x &=8 \end{align}\]

Делим обе части уравнения на 4:

\[\begin{align}4x &=8|:4 \\ x &=2 \end{align}\]

Получили корень уравнения $x=2$.

Пример 7. Решите уравнение: $x-11=x+5$.

Вычитаем из обеих частей $x$ и добавляем 11:

\[\begin{align}x-11 &=x+5|-x+11 \\ 0 &=16 \end{align}\]

Последнее равенство уже не является уравнением. Точнее, является, но это будет уравнение вида $0\cdot x=16$. Коэффициент $a=0$, коэффициент $b=16\ne 0$. Следовательно, корней нет.

При решении настоящих уравнений вовсе не обязательно детально комментировать каждый шаг. Достаточно поставить вертикальную черту справа от уравнения и арифметическими знаками пояснить, что именно вы собираетесь делать.

А в будущем и этих пояснений от вас уже не потребуется.

4. Более сложные соображения

В начале урока мы обнаружили, что далеко не все уравнения сводятся к линейным с помощью элементарных преобразований.

Существует множество способов преобразовать уравнение, но нам пока доступны лишь три элементарных преобразования и ещё вот такая хитрость:

Теорема. Произведение равно нулю тогда и только тогда, когда хотя бы один из множителей равен нулю.

Другими словами, если $a\cdot b=0$, то обязательно либо $a=0$, либо $b=0$.

А это уже интересный приём, который значительно расширяет наши возможности!

Пример 8. Решите уравнение: $\left( 2x-6 \right)\left( x+1 \right)=0$.

Произведение равно нулю, поэтому либо $2x-6=0$, либо $x+1=0$. Получили два линейных уравнения. Решим первое из них:

\[\begin{align}2x-6 &=0 \\ 2x &=6 \\ x &=3 \end{align}\]

Теперь решим второе. Тут вообще всё просто:

\[\begin{align}x+1 &=0 \\ x &=-1 \end{align}\]

Итого уравнение имеет два различных корня: $x=3$ и $x=-1$.

Пример 9. Решите уравнение: $x\left( 5x+15 \right)=0$.

Всё то же самое: произведение равно нулю, поэтому либо $x=0$, либо $5x+15=0$. Первое уравнение уже решено, а второе решается по стандартному алгоритму:

\[\begin{align}5x+15 &=0 \\ 5x &=-15 \\ x &=-3 \end{align}\]

Итого вновь два корня: $x=0$ и $x=-3$.

Разумеется, множителей может быть не два, а три и более. Алгоритм решения от этого никак не меняется: приравнять каждый множитель к нулю и решить каждое полученное уравнение отдельно.

5. Практика

Задача 1

Решите уравнение:

\[6x+72=0\]

Решение. Это линейное уравнение решается через элементарные преобразования:

\[\begin{align}6x+72 &=0 \\ 6x &=-72 \\ x &=-\frac{72}{6} \\ x &=-12 \end{align}\]

Ответ: $x=-12$. Уравнение имеет один корень.

Задача 2

Решите уравнение:

\[5\left( x+9 \right)=5x+45\]

Решение. Сначала раскроем скобки.

Это действие не является элементарным преобразованием уравнений. Оно вообще не относится к уравнениям — оно относился к выражениям с переменной (точнее, как мы позже узнаем, к многочленам):

\[5x+45=5x+45\]

Теперь собираем все слагаемые с переменной $x$ слева, а все числовые слагаемые — справа:

\[\begin{align}5x+45 &=5x+45 \\ 5x-5x &=45-45 \\ 0\cdot x &=0 \end{align}\]

Ответ: все числа. Это уравнение имеет бесконечное множество корней.

Задача 3

Решите уравнение:

\[\left( 6-x \right)+\left( 12+x \right)-\left( 3-2x \right)=15\]

Решение. Вновь сначала раскроем все скобки и упростим полученное выражение:

\[\begin{align}\left( 6-x \right)+\left( 12+x \right)-\left( 3-2x \right) &=15 \\ 6-x+12+x-3+2x &=15 \\ 2x+15 &=15 \end{align}\]

Дальше остаётся лишь выполнить элементарные преобразования:

\[\begin{align}2x &=15-15 \\ 2x &=0 \\ x &=0 \end{align}\]

Ответ: $x=0$. Уравнение имеет единственный корень.

Важное замечание

Линейное уравнение вида $ax+b=0$ требует особого внимания при $a=0$. Потому что делить на ноль нельзя.

Однако если $a\ne 0$, но зато $b=0$, то ничего страшного и «нестандартного» не происходит. Получается уравнение $ax=0$, корнем которого является $x=0$.

Смотрите также:

  1. Иррациональное уравнение: учимся решать методом уединения корня
  2. Что такое дискриминант? И зачем он нужен для решения квадратных уравнений.
  3. Тест на тему «Значащая часть числа»
  4. Иррациональные неравенства. Часть 1
  5. Процент: налоги и зарплата. Считаем с помощью коэффициентов
  6. Более сложные задачи на производительность

Подготовка школьников к ЕГЭ и ОГЭ (Справочник по математике — Алгебра

Справочник по математикеАлгебраУравнения четвертой степени
Схема метода Феррари
Приведение уравнений 4-ой степени
Разложение на множители. Кубическая резольвента
Пример решения уравнения 4-ой степени

Схема метода Феррари

      Целью данного раздела является изложение метода Феррари, с помощью которого можно решать уравнения четвёртой степени

a0x4 + a1x3 + a2x2 +
+ a3x + a4 = 0,
(1)

где a0, a1, a2, a3, a4 – произвольные вещественные числа, причем

      Метод Феррари состоит из двух этапов.

      На первом этапе уравнения вида (1) приводятся к уравнениям четвертой степени, у которых отсутствует член с третьей степенью неизвестного.

      На втором этапе полученные уравнения решаются при помощи разложения на множители, однако для того, чтобы найти требуемое разложение на множители, приходится решать кубические уравнения.

Приведение уравнений 4-ой степени

      Разделим уравнение (1) на старший коэффициент a0 . Тогда оно примет вид

x4 + ax3 + bx2 +
+ cx + d = 0,
(2)

где a, b, c, d – произвольные вещественные числа.

      Сделаем в уравнении (2) замену

(3)

где y – новая переменная.

      Тогда, поскольку

то уравнение (2) принимает вид

В результате уравнение (2) принимает вид

      Если ввести обозначения

то уравнение (4) примет вид

y4 + py2 + qy + r = 0,(5)

где p, q, r – вещественные числа.

      Первый этап метода Феррари завершён.

Разложение на множители. Кубическая резольвента

      Добавив и вычитая в левой части уравнения (5) выражение

2sy2 + s2,

где s – некоторое число, которое мы определим чуть позже, из (5) получим

      Следовательно, уравнение (5) принимает вид

      Если теперь выбрать число s так, чтобы оно являлось каким-нибудь решением уравнения

(7)

то уравнение (6) примет вид

      Избавляясь от знаменателя, уравнение (7) можно переписать в виде

или, раскрыв скобки, — в виде

(9)

      Полученное кубическое уравнение (9), эквивалентное уравнению (7), называют кубической резольвентой уравнения 4-ой степени (5).

      Если какое-нибудь решение кубической резольвенты (9) найдено, то уравнение (8) можно решить, разложив его левую часть на множители с помощью формулы сокращенного умножения «Разность квадратов».

      Действительно,

      Таким образом, для решения уравнения (8) остаётся решить квадратное уравнение

(10)

а также квадратное уравнение

(11)

      Вывод метода Феррари завершен.

Пример решения уравнения 4-ой степени

      Пример. Решить уравнение

x4 + 4x3 – 4x2
– 20x – 5 = 0.
(12)

      Решение. В соответствии с (3) сделаем в уравнении (12) замену

x = y – 1.(13)

      Поскольку

x4 + 4x3 – 4x2 – 20x – 5 =
= (y – 1)4 + 4(y – 1)3
– 4(y – 1)2 – 20(y – 1)– 5 =
= y4 – 4y3 + 6y2 – 4y + 1 +
+ 4y3 – 12y2 + 12y – 4 –
– 4y2 + 8y – 4 –
– 20y + 20 – 5 =
= y4 – 10y2 – 4y + 8,

то в результате замены (13) уравнение (12) принимает вид

y4 – 10y2 – 4y + 8 = 0.(14)

      В соответствии с (5) для коэффициентов уравнения (14) справедливы равенства

p = – 10,      q = – 4,       r = 8.(15)

      В силу (9) и (15) кубической резольвентой для уравнения (14) служит уравнение

2s3 + 10s2 – 16s – 84 = 0,

которое при сокращении на 2 принимает вид:

s3 + 5s2 – 8s – 42 = 0.(16)

      Проверяя, какой из делителей свободного члена уравнения (16) является целым корнем этого уравнения, находим, что целым корнем кубической резольвенты является число

s = – 3.(17)

      Подставляя значения (15) и (17) в формулу (10), получаем уравнение

y2 – 2y – 4 = 0,

корни которого имеют вид:

(18)

      Подставляя значения (15) и (17) в формулу (11), получаем уравнение

y2 + 2y – 2 = 0,

корни которого имеют вид:

(19)

      В завершение, воспользовавшись формулой (13), из (18) и (19) находим корни уравнения (12):

Ответ.

      Замечание. При решении примера мы попутно получили разложение левой части уравнения (14) на множители:

y4 – 10y2 – 4y + 8 =
= (y2 – 2y – 4) (y2 +
+ 2y – 2).
(20)

      Предоставляем посетителю нашего сайта возможность убедиться в справедливости равенства (19) в качестве несложного упражнения.

 

Сообщество Экспонента

  • Публикация
  • 10.09.2022

Системы управления, Электропривод и силовая электроника, Другое

Планирую написать книгу про модельно-ориентированное программирование с автоматическим генерированием кода применительно к разработке разнообразных микропроцессорных систем управления электроприводов. В этой книге в научно-практическо-методической форме я план…

Планирую написать книгу про модельно-ориентированное программирование с автоматическим генерированием кода применительно к разработке разнообразных микропроцессорных систем управления электроприводов.

  • Публикация
  • 24.08.2022

Цифровая обработка сигналов, Системы связи, Математика и статистика

                                                                          &…

Здесь собрана литература по комбинированным методам множественного доступа, в которых используется разделение пользователей в нескольких ресурсных пространствах.

  • вопрос
  • 23.08.2022

Математика и статистика, Радиолокация, Цифровая обработка сигналов

Есть записанный сигнал с датчика (синус с шумом). Как определить соотношение сигнал/шум?

Есть записанный сигнал с датчика (синус с шумом). Как определить соотношение сигнал/шум?

4 Ответа

  • ЦОС
  • цифровая обработка сигналов

23.08.2022

  • Публикация
  • 23.08.2022

Цифровая обработка сигналов, Системы связи, Математика и статистика

                                                                          &. ..

Здесь соборана литература по методам множественного доступа с поляризационным разделением и разделением по орбитальном угловому моменту.

  • Публикация
  • 16.08.2022

Цифровая обработка сигналов, Системы связи, Математика и статистика

                                      

Здесь собрана литература по методам множественного доступа с пространственным разделением.

  • вопрос
  • 22.07.2022

Изображения и видео, Цифровая обработка сигналов, Математика и статистика, Биология, Встраиваемые системы, Глубокое и машинное обучение(ИИ), Автоматизация испытаний, ПЛИС и СнК, Системы управления, Другое

Здравствуйте. Мне нужно обработать большое количество файлов с похожими названиями, каждый блок файлов относится к отдельному объекту, например: file_1_1.txt file_1_2.txt file_1_3.txt file_1_4.txt fil…

Здравствуйте. Мне нужно обработать большое количество файлов с похожими названиями, каждый блок файлов относится к отдельному объекту, например: file_1_1.txt file_1_2.txt file_1_3.txt file_1_4.txt fil…

2 Ответа

  • чтение

22.07.2022

  • вопрос
  • 17.07.2022

Математика и статистика, Цифровая обработка сигналов

Уважаемые коллеги, добрый вечер! В общем, возникла проблема следующего характера. Имеется сигнал, достаточно большой объем точек, длительность порядка 35-40 секунд. Он представлят собой последовательн…

Уважаемые коллеги, добрый вечер! В общем, возникла проблема следующего характера. Имеется сигнал, достаточно большой объем точек, длительность порядка 35-40 секунд. Он представлят собой последовательн…

  • MATLAB
  • Signal Processing

17.07.2022

  • вопрос
  • 15.07.2022

Системы связи, Цифровая обработка сигналов

Здравствуйте! Сделала в симулинке модель сигнала с модуляцией QPSK. На входе сигнала подала последоватльномть с Генератора Бернули бинарного. Sample time: 1/8000. ПРи выводе сигнала на анализатор спек…

Здравствуйте! Сделала в симулинке модель сигнала с модуляцией QPSK. На входе сигнала подала последоватльномть с Генератора Бернули бинарного. Sample time: 1/8000. ПРи выводе сигнала на анализатор спек…

  • сигнал
  • модуляция
  • qpsk
  • скорость бита
  • битрейт
  • символьная скорость
  • скорость передачи информации
  • цифровая манипуляция

15.07.2022

  • Публикация
  • 13.07.2022

Цифровая обработка сигналов, Системы связи, Математика и статистика

                                                                          &…

Здесь собрана литература по методам множественного доступа с кодовым разделением

  • Публикация
  • 12. 07.2022

Цифровая обработка сигналов, Системы связи, Математика и статистика

                                                                          &…

Здесь собрана литература по методам множественного доступа с разделением по мощности.

Результаты поиска

Нет результатов поиска, попробуйте задать другие параметры.

Примеры решения показательных уравнений

Примеры решения показательных уравнений

Пример №1

1000x=100

Представим левую и правую часть уравнения в виде степени, имеющую одинаковые основания:

103x=102

Теперь, когда основания одинаковые, нужно приравнять показатели степеней.

3x=2
x=2/3

Ответ: x=2/3 .

Главное в показательных уравнениях — свести левую и правую часть уравнения к общему основанию:

Пример №2

(2/5)x=(5/2)4

Представим (2/5)x как (5/2)-x:

(5/2)-x=(5/2)4

Основания одинаковые, следовательно, приравниваем показатели:

-x=4
x=-4

Ответ: x=-4

Пример №3

√3х=9

√3х распишем как 3x/2, а 9 — как 32:

3х/2=32

Приравниваем показатели:

х/2=2
х=4

Ответ: x=4

Пример №4

3х2-х-2=81

Заметим, что 81=34

3х2-х-2=34

Приравниваем показатели:

х2-х-2=4

х2-х-6=0

Получили квадратное уравнение:

D=1+24=25, D>0, следовательно, уравнение имеет два действительных корня

х1=(1+5)/2=3

х2=(1-5)/2=-2

Ответ: х=3 и х=-2

Пример №5

4х+1+4х=320

В таких случаях выносится основание с наименьшим показателем. В данном уравнении наименьшим показателем является х. Вынесем 4х за скобки:

4х(4+1)=320

4х*5=320

Представим 320 в виде 5*43, тогда:

4х*5=5*43

Поделим левую и правую часть уравнения на 5:

4х=43

Приравняем показатели:

х=3

Ответ: х=3

Пример №6

7х+2+4*7х-1=347

Степенью с наименьшим показателем в этом уравнении является х-1, следовательно, за скобки выносим 7x-1. Получаем:

7х-1*(73+4)=347

7х-1*347=347

Поделим левую и правую часть уравнения на 347:

7х-1=1

Заметим, что любое число в нулевой степени равно 1. Следовательно, распишем 1 как 70:

7х-1=70

Приравняв показатели, получим:

х-1=0

х=1

Ответ: х=1

Пример №7

4х-5*2х+4=0

Представим 4х как 2, получим:

2-5*2х+4=0

Введем подстановку: 2х обозначим переменной t. Cледовательно: 2=t2. Получим:

t2-5t+4=0

Найдем корни уравнения по теореме Виета:

t1=1

t2=4

Заменим t на 2х:

2х=1

Заметим, что 20=1

2х=20

Приравняем показатели:

х=0

2х=4

Заметим, что 4=22

2х=22

Приравняем показатели:

х=2

Уравнение имеет два действительных корня 0 и 2.

Ответ: х=0 и х=2

Пример №8

(√2+√3)х + (√2-√3)х=4

Введем подстановку: (√2+√3)х обозначим переменной t. А (√2-√3)х домножим на сопряженные и получим:

((√2+√3)х*(√2-√3)х) / (√2+√3)х = (√4-3)х/(√2+√3)х = 1 x/(2+√3)x = 1/(2+√3)x

Следовательно, 1/(√2+√3)х=1/t.

Получаем:

t+1/t=4

Отметим, что t=0, т.к. деление на 0 не определено. Домножим левую и правую часть на t:

t2+1=4t

t2-4t+1=0

Решим квадратное уравнение:

D=16-4=12, D>0, следовательно, уравнение имеет два действительных корня

t1=(4-2√3)/2=2-√3

t2=(4+2√3)/2=2+√3

Заменим t на (√2+√3)х:

(√2-√3)х=2+√3

Домножим 2+√3 на сопряженные и получим:

1/(2-√3)=2+√3

Cледовательно:

(√2-√3)х=1/2-√3

Заметим, что 1/2-√3=(√2-√3)-2

(√2+√3)х=(√2-√3)-2

Приравняв показатели, получим:

х=-2

Заменим t на 2+√3

(√2+√3)х=2+√3

Заметим, что 2+√3=(√2+√3)2

Приравняв показатели, получим:

х=2

Ответ: х=-2 и х=2

Пример №9

x+y=6

xy2+7y+12=1

Выразим x:

x=6-y

xy2+7y+12=1

Заметим, что x0=1:

x=6-y

xy2+7y+12=x0

Приравним показатели:

x=6-y

y2+7y+12=0

Решим отдельно квадратное уравнение:

y2+7y+12=0

D=49-48=1, D>0, следовательно, уравнение имеет два действительных корня

y1=(-7+1)=-3

y2=(-7-1)=-4

y=-3

x=6-(-3)=9

y=-4

x=6-(-4)=10

Ответ: x=9; y=-3 и x=10; y=-4

<< Назад ] [ Начало ] [ Вперед >>


Решение линейных уравнений с примерами

Уравнение с одним неизвестным, которое после раскрытия скобок и приведения подобных членов принимает вид  

aх + b = 0, где a и b произвольные числа, называется линейным уравнением с одним неизвестным. Cегодня разберёмся, как эти линейные уравнения решать.

Например, все уравнения:

2х + 3= 7 – 0,5х;  0,3х = 0;  x/2 + 3 = 1/2 (х – 2) — линейные.

Значение неизвестного, обращающее уравнение в верное равенство называется решением или корнем уравнения.

Например, если в уравнении 3х + 7 = 13 вместо неизвестного х подставить число 2 , то получим верное равенство 3· 2 +7 = 13. Значит, значение х = 2 есть решение или корень уравнения.

А значение х = 3 не обращает  уравнение  3х + 7 = 13 в верное равенство, так  как  3· 2 +7 ≠ 13. Значит, значение х = 3 не является решением или корнем уравнения.

Решение любых линейных уравнений сводится к решению уравнений вида

aх + b = 0.

Перенесем свободный член из левой части уравнения в правую, изменив при этом знак перед b на противоположный, получим

aх = ‒ b.

Если a ≠ 0, то х = ‒ b/a .

Пример 1. Решите уравнение 3х + 2 =11.

Перенесем 2 из левой части уравнения в правую, изменив при этом знак перед 2 на противоположный, получим 
3х = 11 – 2.

Выполним вычитание, тогда
3х = 9.

Чтобы найти х надо разделить произведение на известный множитель, то есть     
х = 9 : 3.

Значит, значение х = 3 является  решением или корнем уравнения.

Ответ: х = 3.

Если а = 0 и b = 0, то получим уравнение  0х = 0. Это уравнение имеет бесконечно много  решений, так как при умножении любого числа на 0 мы получаем 0,но b тоже равно 0. Решением этого уравнения  является любое число.

Пример 2. Решите уравнение 5(х – 3) + 2 = 3 (х – 4) + 2х ‒ 1.

Раскроем скобки:
5х – 15 + 2 = 3х – 12 + 2х ‒ 1.

Сгруппируем  в левой части члены, содержащие неизвестные, а в правой ‒ свободные члены:
5х – 3х ‒ 2х =  – 12  ‒ 1 + 15 ‒ 2.

Приведем подобные члены:
0х = 0.

Ответ: х —  любое число.

Если а = 0 и b ≠ 0, то получим уравнение  0х = — b. Это уравнение решений не имеет, так как при умножении любого числа на 0 мы получаем 0, но  b ≠ 0 .

Пример 3. Решите уравнение х + 8 = х + 5.

Сгруппируем  в левой части члены, содержащие неизвестные, а в правой ‒ свободные члены:
х – х = 5 ‒ 8.

Приведем подобные члены: 
0х = ‒ 3.

Ответ: нет решений.

На рисунке 1 изображена схема решения линейного уравнения

Составим общую схему решения уравнений с одной переменной. Рассмотрим решение примера 4.

Пример 4. Пусть надо решить уравнение 

1) Умножим все члены уравнения на наименьшее общее кратное знаменателей, равное 12.

2) После сокращения получим
4 (х – 4) + 3·2 (х + 1) ‒ 12 = 6·5 (х – 3) + 24х – 2 (11х + 43)

3) Чтобы отделить члены, содержащие неизвестные и свободные члены, раскроем скобки:
4х – 16 + 6х + 6 – 12 = 30х – 90 + 24х – 22х – 86 .

4) Сгруппируем в одной части члены, содержащие неизвестные, а в другой – свободные члены:
4х + 6х – 30х – 24х + 22х = ‒ 90 – 86 + 16 – 6 + 12.

5) Приведем подобные члены:
‒ 22х = ‒ 154.

6) Разделим на  – 22 , Получим
х = 7.

Как видим, корень уравнения равен семи.

Вообще такие уравнения можно решать по следующей схеме:

а) привести уравнение к целому виду;

б) раскрыть скобки;

в) сгруппировать члены, содержащие неизвестное, в одной части уравнения, а свободные члены ‒ в другой;

г) привести подобные члены;

д) решить уравнение вида aх = b,которое получили после приведения подобных членов.

Однако эта схема не обязательна для всякого уравнения. При решении многих более простых уравнений приходится начинать не с первого, а со второго (Пример. 2),  третьего (Пример. 1, 3) и даже с пятого этапа, как в примере 5.

СЛОЖНА-А-А 🙀 Ты же знаешь, что если не разобраться в теме сейчас, то потом придется исправлять оценки. Беги на бесплатное онлайн-занятие с репетитором (подробности тут + 🎁).

Пример 5. Решите уравнение 2х = 1/4.

Находим неизвестное  х = 1/4 : 2,
х = 1/8
.

Рассмотрим решение некоторых линейных уравнений, встречающихся на основном государственном экзамене.

Пример 6. Решите уравнение 2 (х + 3) = 5 – 6х.

Решение

2х + 6 = 5 – 6х

2х + 6х = 5 – 6

8х = ‒1

х = ‒1 : 8

х = ‒ 0, 125

Ответ: ‒ 0, 125

Пример 7. Решите уравнение – 6 (5 – 3х) = 8х – 7.

Решение

– 30 + 18х = 8х – 7

18х  – 8х =  – 7 +30

10х = 23

х = 23 : 10

х = 2,3

Ответ: 2,3

Пример 8. Решите уравнение

 

Решение:

3(3х – 4) = 4 · 7х + 24

9х – 12 = 28х + 24

9х – 28х = 24 + 12

-19х = 36

х = 36 : (-19)

х = — 36/19

Ответ: — .  

Пример 9. Найдите f(6), если f (x + 2) = 37-х

Решение

Так как надо найти f(6), а нам известно f (x + 2),
то х + 2 = 6.

Решаем линейное уравнение х + 2 = 6,
получаем х = 6 – 2, х = 4.

Если х = 4, тогда
f(6) = 37-4 = 33 = 27

Ответ: 27.

Молодец! Раз ты дочитал это до конца, вероятно, ты все отлично усвоил.  Но если вдруг что-то еще непонятно — попробуй онлайн-занятие с репетитором (подробности тут + 🎁).

Если у Вас остались вопросы, есть желание разобраться с решением уравнений более основательно, записывайтесь на мои уроки в РАСПИСАНИИ. Буду рада Вам помочь!

Также TutorOnline советует посмотреть новый видеоурок от нашего репетитора Ольги Александровны, который поможет разобраться как с линейными уравнениями, так и с другими.

 

© blog. tutoronline.ru, при полном или частичном копировании материала ссылка на первоисточник обязательна.

20 примеров линейных уравнений с ответами

Линейные уравнения можно решать, применяя различные операции к обеим сторонам знака равенства. Эти операции могут помочь нам упростить уравнение, найти переменную и, в конечном итоге, найти решение.

В этой статье мы рассмотрим краткое изложение линейных уравнений, а затем 20 примеров с ответами, чтобы освоить процесс решения уравнений первой степени.

АЛГЕБРА

Относится к

Обучение решению линейных уравнений на примерах.

См. примеры

Содержание

АЛГЕБРА

Актуально для

Обучение решению линейных уравнений на примерах.

См. примеры

Как решать линейные уравнения?

Напомним, что линейные уравнения — это уравнения, в которых все переменные имеют максимальную степень 1. Например, уравнения $latex 4x+1=5$ и $latex 2x+12=4x-2$ являются линейными уравнениями.

Чтобы решить линейные уравнения, мы должны применить различные операции к обеим сторонам знака равенства, чтобы мы могли найти переменную. Таким образом, мы можем выполнить следующие шаги, чтобы найти решение линейных уравнений:

Шаг 1:  Мы упрощаем выражение. Это включает в себя удаление круглых скобок и других знаков группировки, удаление дробей и объединение подобных терминов.

Шаг 2:  Изолируем переменную. Мы выполняем сложение и вычитание, чтобы поместить все члены с переменными только в одну сторону уравнения.

Шаг 3: Решаем уравнение. Мы делаем умножение и деление, чтобы найти ответ.


20 примеров линейных уравнений с ответами

Следующие 20 примеров линейных уравнений имеют соответствующие решения, где процесс указан шаг за шагом. Рекомендуется попробовать решить примеры самостоятельно, прежде чем смотреть ответ.

ПРИМЕР 1

Решите уравнение $латекс 5x-12=3$.

Решение

Шаг 1:  Упрощение: здесь нам нечего упрощать.

Шаг 2:  Решение для переменной: Мы используем сложение для решения переменной:

$latex 5x-12=3$

$latex 5x-12+12=3+12$

$latex 5x =15$

Шаг 3:  Решение: Делим обе части на 5:

$latex \frac{5x}{5}=\frac{15}{5}$

$latex x=3$

ПРИМЕР  2

Решите уравнение $латекс 3x+1=x-3$.

Решение

Шаг 1:  Упрощение: Нам нечего упрощать.

Шаг 2:  Найти переменную: мы используем сложение и вычитание, чтобы найти переменную:

$latex 3x+1=x-3$

$latex 3x+1-1=x-3-1 $

$latex 3x=x-4$

$latex 3x-x=x-4-x$

$latex 2x=-4$

Шаг 3:  Решение: делим обе части на 2:

$латекс \frac{2x}{2}=\frac{-4}{2}$

$латекс x=-2$

ПРИМЕР  3

Найдите значение  t  в уравнении $latex 5t+5=3t+7$.

Решение

Шаг 1:  Упрощение: у нас нет похожих терминов.

Шаг 2:  Найти переменную: мы используем вычитание для нахождения переменной:

$latex 5t+5=3t+7$

$latex 5t+5-5=3t+7-5$

$латекс 5т=3т+2$

$латекс 5т-3т=3т+2-3т$

$латекс 2т=2$

Шаг 3:  Решение: делим обе части на 2:

$latex \frac{2t}{2}=\frac{2}{2}$

$latex t=1$

Начните сейчас: изучить наши дополнительные математические ресурсы

ПРИМЕР  4

Решите уравнение $латекс 3(2x+1)=-9$.

Решение

Шаг 1:  Упрощение: Раскрываем скобки:

$latex 3(2x+1)=-9$

$latex 6x+3=-9$

Шаг 2: 0 0090 для переменной: мы используем вычитание для решения для переменной:

$латекс 6x+3=-9$

$латекс 6x+3-3=-9-3$

$латекс 6x=-12$

Шаг 3:  Решение: делим обе части на 6 :

$latex \frac{6x}{6}=\frac{-12}{6}$

$latex x=-2$

ПРИМЕР  5

Решите уравнение $latex 2( 2x-5)=3(x-1)-4$.

Решение

Шаг 1:  Упростите: раскроем круглые скобки с обеих сторон уравнения и объединим одинаковые члены:

$латекс 2(2x-5)=3(x-1)-4$

$latex 4x-10=3x-3-4$

$latex 4x-10=3x-7$

Шаг 2:  Найти переменную: мы используем сложение и вычитание, чтобы найти переменную:

$латекс 4x-10+10=3x-7+10$

$латекс 4x=3x+3$

$латекс 4x-3x=3-3x$

$латекс x=3$

Шаг 3 .0140 z  в уравнении $латекс 3(z-2)+10=2(2z+2)+2$.

Решение

Шаг 1:  Упрощение: Раскрываем скобки и объединяем одинаковые термины:

$latex 3(z-2)+10=2(2z+2)+2$

$latex 3z-6 +10=4z+4+2$

$latex 3z+4=4z+6$

Шаг 2:  Найти переменную: мы используем вычитание, чтобы найти переменную:

$latex 3z+4- 4=4z+6-4$

$латекс 3z=4z+2$

$латекс 3z-4z=2$

$латекс -z=2$

Шаг 3:  Решение: Делим обе части на -1:

$latex \frac{-z}{-1}=\frac{2}{-1}$

$latex z=-2 $

ПРИМЕР  7

Решите уравнение $latex \frac{2x+1}{3}=x-1$.

Решение

Шаг 1:  Упрощение: умножаем на 3, чтобы исключить дробь:

$latex \frac{2x+1}{3}=x-1$

$latex 2x+1=3x- 3$

Шаг 2:  Решите для переменной: мы вычитаем 1 и 3x с обеих сторон:

$латекс 2x+1=3x-3$

$латекс 2x+1-1=3x-3-1$

$латекс 2x=3x-4$

$латекс 2x-3x=3x-4- 3x$

$latex -x=-4$

Шаг 3:  Решение: делим обе части на -1:

$latex \frac{-x}{-1}=\frac{-4} {-1}$

$latex x=4$

ПРИМЕР  8

Решите уравнение $latex \frac{4x}{3}-2= \frac{2x+3}{3} — 1$.

Решение

Шаг 1:  Упрощение: мы умножаем обе части уравнения на 3, чтобы исключить дроби и объединить одинаковые члены:

$латекс \frac{4x}{3}-2=\frac{2x+3}{3}-1$.

$latex 4x-6=2x+3-3$

$latex 4x-6=2x$

Шаг 2:  Решение для переменной: мы добавляем 6 и вычитаем 2x с обеих сторон:

$latex 4x-6+6=2x+6$

$латекс 4x=2x+6$

$латекс 4x-2x=2x+6-2x$

$латекс 2x=6$

Шаг 3: Решить : Делим обе части на 2:

$latex \frac{2x}{2}=\frac{6}{2}$

$latex x=3$

ПРИМЕР  9

Найдите значение t в уравнении $latex \frac{2t-5}{5}+2=\frac{t-2}{3}+2$.

Решение

Шаг 1:  Упрощение: мы умножаем на 15, чтобы исключить дроби и объединить одинаковые члены:

$latex \frac{2t-5}{5}+2=\frac{t-2}{ 3}+2$.

$3(2t-5)+15(2)=5(t-2)+15(2)$$

$латекс 6т-15+30=5т-10+30$

$латекс 6т+ 15=5t+20$

Шаг 2:  Решение для переменной: мы вычитаем 15 и 5 t с обеих сторон:

$латекс 6т+15=5т+20$

$латекс 6т+15-15=5т+20-15$

$латекс 6т=5т+5$

$латекс 6т -5t=5t+5-5t$

$латекс t=5$

Шаг 3: Решите: нам больше не нужно делить:

$латекс t=5$

ПРИМЕР 070 1

9

Решите уравнение $latex \frac{2x-3}{x+1}+2=3$.

Решение

Шаг 1:  Упрощение: мы умножаем обе части на ( x +1) и объединить подобные термины:

$latex \frac{2x-3}{x+1}+2=3$

$latex 2x-3+2(x+1)=3(x+1 )$

$latex 2x-3+2x+2=3x+3$

$latex 4x-1=3x+3$

Шаг 2:  Решение для переменной: добавьте 1 и вычтите 3  x с двух сторон:

$латекс 4x-1=3x+3$

$латекс 4x-1+1=3x+3+1$

$латекс 4x=3x+4$

$латекс 4x-3x= 3x+4-3x$

$латекс x=4$

Шаг 3:  Решение: нам больше не нужно делить:

$латекс x=4$

ПРИМЕР  11

Найдите значение  t  в уравнении $латекс 3t+4(t-10)=t+20$.

Решение

Шаг 1:  Упрощение: Раскрываем скобки и объединяем одинаковые термины:

$latex 3t+4(t-10)=t+20$

$latex 3t+4t-40=t+ 20$

$латекс 7t-40=t+20$

Шаг 2: Решить для переменной: мы добавляем 40 и вычитаем t с обеих сторон:

$латекс 7t-40=t+20$

$латекс 7t-40+40=t+20+40$

$латекс 7t=t+60$

$латекс 7t-t=t+60- t$

$latex 6t=60$

Шаг 3:  Решение: Делим обе части на 6:

$latex \frac{6t}{6}=\frac{60}{6}$

$latex t=10$

ПРИМЕР  12

Решите уравнение $latex 3x+6(x+1)=3(x+1)+5$.

Решение

Шаг 1:  Упрощение: мы раскрываем скобки и объединяем подобные термины:

$латекс 3x+6(x+1)=3(x+1)+5$

$латекс 3x+6x+6=3x+3+5$

$латекс 9x+6=3x+8$

Шаг 2:  Решите для переменной: мы вычитаем 6 и 3  x  с обеих сторон:

$latex 9x+6=3x+8$

$latex 9x+6-6=3x+8- 6$

$латекс 9x=3x+2$

$латекс 9x-3x=3x+2-3x$

$латекс 6x=2$

Шаг 3: Решить: Делим обе части на 6:

$latex \frac{6x}{6}=\frac{2}{6}$

$latex x=\frac{1}{3}$

ПРИМЕР  13

Найдите значение  x  в уравнении $latex \frac{1}{x+2}+2=\frac{9}{4}$.

Решение

Шаг 1:  Упростим: умножаем все уравнение на 4 ( x +2) и объединяем одинаковые члены:

$latex \frac{1}{x+2}+2=\frac {9}{4}$

$латекс 4+8(x+2)=9(x+2)$

$латекс 4+8x+16=9x+18$

$латекс 8x+20=9x +18$

Шаг 2:  Решение для переменной: мы вычитаем 20 и 9 x с обеих сторон:

$латекс 8x+20-20=9x+18-20$

$латекс 8x=9x-2$

$латекс 8x-9x=9x-2-9x$

$ латекс -x=-2$

Шаг 3:  Решение: Делим обе части на -1:

$latex \frac{-x}{-1}=\frac{-2}{-1}$

$latex x=2$

ПРИМЕР  14

Найдите значение  y  в уравнении $$2y+3(2y-5)+4=y+3(2y-2)- 5$$

Решение

Шаг 1:  Упрощение: раскрываем круглые скобки и объединяем одинаковые термины:

$$2y+3(2y-5)+4=y+3(2y-2)-6$$

$latex 2y+6y-15+4 =y+6y-6-6$

$latex 8y-11=7y-12$

Шаг 2: Решите для переменной: мы прибавляем 11 и вычитаем 7 y с обеих сторон:

$latex 8y-11=7y-12$

$латекс 8y-11+11=7y-12+11$

$латекс 8y=7y-1$

$латекс 8y-7y=7y-1-7y$

$latex y=-1$

Шаг 3:  Решение: нам больше не нужно делить:

$latex y=-1$

ПРИМЕР  15

Решите уравнение $latex \frac{4x-9}{3}+2=3(x-2)$.

Решение

Шаг 1:  Упрощение: умножаем все уравнение на 3, раскрываем скобки и объединяем одинаковые члены:

$latex \frac{4x-9}{3}+2=3(x- 2)$

$латекс 4x-9+6=9(x-2)$

$латекс 4x-3=9x-18$

Шаг 2: Решить для переменной: мы добавляем 3 и вычитаем 9 90 140 x 90 141  с обеих сторон:

$латекс 4x-3+3=9x-18+3$

$латекс 4x=9x-15$

$латекс 4x-9x=9x-15-9x$

$латекс -5x=-15$

Шаг 3:  Решение: Делим обе части на -5:

$latex \frac{-5x}{-5}=\frac{-15}{-5}$

$latex x=3 $

→ Калькулятор линейных уравнений

ПРИМЕР  16

Найдите значение  x  в уравнении $latex -3x+18=-x(13-10)+4x-2$.

Раствор

Шаг 1:  Упрощение: мы раскрываем круглые скобки и объединяем одинаковые термины:

$латекс -3x+18=-x(13-10)+4x-2$

$латекс -3x+18=-x(3 )+4x-2$

$latex -3x+18=-3x+4x-2$

$latex -3x+18=x-2$

Шаг 2:  Найти переменную: вычесть 18 и вычтите x  с обеих сторон:

$латекс -3x+18-18=x-2-18$

$латекс -3x=x-20$

$латекс -3x-x=x-20- x$

$латекс -4x=-20$

Шаг 3:  Решение: делим обе части на -4:

$latex \frac{-4x}{-4}=\frac{-20}{-4}$

$latex x=5$

ПРИМЕР  17

Найдите значение  w  в уравнении $латекс 10(2w-5)=2w+2(w+1)$.

Решение

Шаг 1:  Упрощение: Раскрываем скобки и объединяем одинаковые термины:

$latex 10(2w-5)=2w+2(w+1)$

$latex 20w-50=2w +2w+2$

$латекс 20w-50=4w+2$

Шаг 2: Решите для переменной: мы добавляем 50 и вычитаем 4  w  с обеих сторон:

$latex 20w-50=4w+2$

$latex 20w-50+50=4w+2+ 50$

$latex 20w=4w+52$

$latex 20w-4w=4w+52-4w$

$latex 16w=52$

Шаг 3:  Решаем: Делим обе части на 16 упростите дробь:

$latex \frac{16w}{16}=\frac{52}{16}$

$latex x=\frac{13}{4}$

ПРИМЕР  18

Найдите значение r  в уравнении $latex 3(-2r-5)+4=\frac{r}{2}+2$.

Решение

Шаг 1:  Упрощение: мы умножаем обе части на 2, чтобы исключить дробь, раскрываем круглые скобки и объединяем одинаковые члены:

$latex 3(-2r-5)+4=\frac{r {2}+2$

$латекс 6(-2r-5)+8=r+4$

$латекс -12r-30+8=r+4$

$латекс -12r-22=r +4$

Шаг 2:  Решение для переменной: мы добавляем 22 и вычитаем r с двух сторон:

$латекс -12r-22=r+4$

$латекс -12r-22+22=r+4+22$

$латекс -12r=r+26$

$latex -12r-r=r+26-r$

$latex -13r=26$

Шаг 3:  Решение: делим обе части на -13:

$latex \frac{-13r}{ -13}=\frac{26}{-13}$

$latex x=-2$

ПРИМЕР  19

Найдите значение  x  в уравнении $$3x+4(- 2x+1)=3(x-5)+2(2x-7)-3$$

Решение

Шаг 1:  Упрощение: мы раскрываем все скобки и объединяем одинаковые члены:

$$3x+4(-2x+1)=3(x-5)+2(2x-7)-3 $$

$$ 3x-8x+4=3x-15+4x-14-3$$

$latex -5x+4=7x-32$

Шаг 2:  Найти переменную: мы вычитаем 4 и 7 x с обеих сторон:

$латекс -5x+4-4=7x-32-4$

$латекс -5x=7x-36$

$латекс -5x-7x=7x-36 -7x$

$латекс -12x=-36$

Шаг 3:  Решение: делим обе части на -12:

$latex \frac{-12x}{-12}=\frac{-36}{-12}$

$latex x=3$

ПРИМЕР  20

Найдите значение x  в уравнении $latex 2\left(\frac{x+2}{4} \right)+2=\frac{3x}{4}+2$.

Решение

Шаг 1:  Упрощение: мы начинаем с упрощения дроби, затем умножаем на 4, чтобы исключить дроби и комбинируем одинаковые члены:

$latex 2\left( \frac{x+2}{4} \right)+2=\frac{3x}{4}+2$

$latex \frac{x+2}{2}+2=\frac{3x}{4}+2$

$latex 2(x+2)+8=3x+8$

$latex 2x +4+8=3x+8$

$latex 2x+12=3x+8$

Шаг 2:  Решение для переменной: мы вычитаем 12 и 3 x  из обеих сторон:

$latex 2x +12-12=3x+8-12$

$латекс 2x=3x-4$

$латекс 2x-3x=3x-4-3x$

$латекс -x=-4$

Шаг 3 :  Решение: делим обе части на -1:

$latex \frac{-x}{-1}=\frac{-4}{-1}$

$latex x=4$


См. также

Хотите узнать больше о решении уравнений? Взгляните на эти страницы:

  • Калькулятор линейных уравнений
  • Упражнения для уравнений первой степени
  • Уравнения первой степени с двумя неизвестными Упражнения
  • Уравнения первой степени с дробями Упражнения
  • Упражнения для решения линейных уравнений
44 дополнительные ресурсы по разным темам

УЗНАТЬ БОЛЬШЕ

сообщите об этом объявлении

Решение уравнений – методы и примеры

Понимание того, как решать уравнения, является одним из самых фундаментальных навыков, которым может овладеть каждый студент, изучающий алгебру. Решения для большинства алгебраических выражений ищутся с применением этого навыка. Поэтому студенты должны стать более опытными в том, как проводить операцию.

Эта статья научит решать уравнение , выполняя четыре основные математические операции: сложение , вычитание , умножение и деление .

Уравнение обычно состоит из двух выражений, разделенных знаком, указывающим на их взаимосвязь. Выражения в уравнении могут быть связаны знаком равенства со знаком (=), меньше (<), больше (>) или комбинацией этих знаков.

Как решать уравнения?

Решение алгебраического уравнения обычно представляет собой процедуру манипулирования уравнением. Переменная остается с одной стороны, а все остальное с другой стороны уравнения.

Проще говоря, чтобы решить уравнение, нужно изолировать, приравняв его коэффициент к 1. Что бы вы ни делали с одной частью уравнения, сделайте то же самое с противоположной частью уравнения.

Решите уравнения, добавив

Давайте рассмотрим несколько примеров ниже, чтобы понять эту концепцию.

Пример 1

Решение: –7 — x =

Решение

–7 — x =

Добавить 7 к обоим сторонам уравнения.
7 — x + 7 = 9 + 7
— x = 16

Умножение обеих сторон на –1
x = –16

Пример 2

Solve 4 = X — 3

Решение

Здесь переменная находится в правой части уравнения. Добавьте 3 к обеим частям уравнения

4+ 3 = x – 3 + 3

7 = x

Проверьте решение, подставив ответ в исходное уравнение.

4 = х – 3

4 = 7 – 3

Следовательно, x = 7 – правильный ответ.

Решение уравнений путем вычитания

Давайте рассмотрим несколько примеров ниже, чтобы понять эту концепцию.

Пример 3

Решение для x in x+ 10 = 16

Решение

x+ 10 = 16

Субтракт 7 из обоих сборов уравнения.

x + 10 – 10 = 16 – 10

x = 6

Пример 4

Решить линейное уравнение 15 = 26 – y

Решение

15 = 26 – y

Вычесть 26 из обеих частей уравнения
= 15 – 9y 9 = -026 – 026 y

Умножьте обе части на –1

y = 11

Решение уравнений с переменными в обеих частях путем сложения

Давайте рассмотрим несколько примеров ниже, чтобы понять эту концепцию.

Пример 4

Рассмотрим уравнение 4x –12 = -x + 8.

Поскольку уравнение имеет две стороны, вам нужно выполнить одну и ту же операцию с обеих сторон.

Добавьте переменную x к обеим частям уравнения

⟹ 4x –12 + x = -x + 8 + x.

Упростить

Упростить уравнение, собрав одинаковые члены с обеих сторон уравнения.

5x – 12 = 8.

Теперь уравнение имеет только одну переменную с одной стороны.

Добавьте константу 12 к обеим частям уравнения.

Константа, прикрепленная к переменной, добавляется с обеих сторон.

⟹ 5x – 12 +12 = 8 + 12

Упростить

Упростить уравнение, объединив одинаковые члены. А 12.

⟹ 5x = 20

Теперь делим на коэффициент.

Деление обеих частей на коэффициент — это просто полное деление на число, прикрепленное к переменной.

Решение этого уравнения равно, следовательно,

x = 4.

Проверьте свое решение

Проверьте правильность решения, подставив ответ в исходное уравнение.

4x –12 = -x + 8

⟹ 4(4) –12 = -4 + 8

4 = 4

Следовательно, решение верное.

Пример 5

Решение -12x -5 -9 + 4x = 8x -13x + 15 -8

Раствор

Упрощайте на комбинациях, такие как термины

-8 -8X -124x -100003

. = -5x +7

Добавьте 5x с обеих сторон.

-8x + 5x -14 = -5x +5x + 7

-3w -14=7

Теперь прибавьте 14 к обеим частям уравнения.

– 3x – 14 + 14 = 7 + 14

-3x = 21

Разделить обе части уравнения на -3

-3x/-3 = 21/3

x = 7.

Решение уравнений с переменными с обеих сторон путем вычитания

Давайте рассмотрим несколько примеров ниже, чтобы понять эту концепцию. Пример 6

Решите уравнение 12x + 3 = 4x + 15

12x-4x + 3 = 4x – 4x + 15

6x + 3= 15

Вычесть константу 3 с обеих сторон.

6x + 3 -3 = 15 – 3

6x = 12

Разделить на 6;

6x/6 = 12/6

x = 2

Пример 7

Решение уравнения 2x — 10 = 4x + 30.

Раствор

Подтех .

2x -2x -10 = 4x – 2x + 23

-10 = 2x + 30

Вычтите обе части уравнения на константу 30.

-10 – 30 = 2x + 30 – 30

– 40 = 2x

Теперь разделите на 2

-40/2 = 2x/2

-20 = x

Решение линейных уравнений с умножением

 

Линейные уравнения решаются с помощью умножения, если при записи уравнения используется деление. Как только вы заметите, что переменная делится, вы можете использовать умножение для решения уравнений.

Пример 7

Решение x/4 = 8

Решение

Умножание обеих сторон уравнения по номинателю фракции,

4 (x/4) = 8 x 4.

x = 32

Пример 8

Решение -x/5 = 9

Решение

Умножение с обеих сторон на 5.

5 (-x/5) =

-x = 45

Умножьте обе части на -1, чтобы сделать коэффициент при переменной положительным.

x = – 45

Решение линейных уравнений с делением

Для решения линейных уравнений с делением обе части уравнения делятся на коэффициент переменной. Давайте посмотрим на примеры ниже.

Пример 9

Решите 2x = 4

Решение

Чтобы решить это уравнение, разделите обе части на коэффициент переменной.

2x/2 = 4/2

x = 2

 

Пример 10

Решение уравнения -2x = −8

Решение

Разделение обеих сторон уравнения на 2.

–2x/2 = –8.

–2x/2 = –8.

= − 4

Умножая обе части на -1, мы получаем;

x = 4

Как решать алгебраические уравнения, используя распределительное свойство?

 

Решение уравнений с использованием распределительного свойства влечет за собой умножение числа на выражение в скобках. Затем сходные термины объединяются, а затем изолируется переменная.

Пример 11

Решить 2x – 2(3x – 2) = 2(x –2) + 20 + 20

Использовать распределительное свойство для удаления скобок
2x – 6x + 4 = 2x – 4 + 20
– 4x + 4 = 2x + 16

Сложение или вычитание с обеих сторон

–4x + 4 – 4 –2x = 2x + 16 – 4 –2x
–6x = 12
x = –2

Проверьте ответ, подставив решение в уравнение.

2x – 2(3x – 2) = 2(x –2) + 20

(2 * –2) – 2((3 * –2) –2) = 2(–2 –2) + 20
12 = 12

 

Пример 12 9000 Решить для x в уравнении -3x – 32 = -2(5 – 4x)

Решение

Примените распределительное свойство, чтобы убрать скобки.

–3x – 32 = – 10 + 8x

Сложение обеих частей уравнения в 3x дает

-3x + 3x – 32 = – 10 + 8x + 3x Добавьте обе части уравнения на 10.

– 10 + 10 + 11x = -32 + 10

11x = -2

Разделите все уравнение на 11.

11x/11 = -22/11

x= -2

9003 с дробями?

Не паникуйте, когда видите дроби в алгебраическом уравнении. Если вы знаете все правила сложения, вычитания, умножения и деления, это для вас пустяк.

Чтобы решить уравнения с дробями, нужно преобразовать их в уравнение без дробей.

Этот метод также называется « очистка дробей ».

При решении уравнений с дробями выполняются следующие шаги:

  • Определите наименьшее общее кратное знаменателей (НОК) всех дробей в уравнении и умножьте на все дроби в уравнении.
  • Изолировать переменную.
  • Упростите обе части уравнения, применяя простые алгебраические операции.
  • Применение свойства деления или умножения, чтобы сделать коэффициент переменной равным 1.

Пример 13

Решить (3x + 4)/5 = (2x – 3)/3

Решение

ЖКД 5 и 3 умножить, следовательно, 39x 90, 8 и 3 15 4)/5 = (2x – 3)/3

{(3x + 4)/5}15 = {(2x – 3)/3}15

9x +12 = 10x -15

Изолировать переменную;

9x -10x = -15-12

-x = -25

x = 25

Пример 14

Solve для x 3/2x + 6/4 = 10/3.

Решение

LCD 2x, 4 и 3 равно 12x

Умножьте каждую дробь в уравнении на LCD.

(3/2x)12x + (6/4)12x = (10/3)12x

=> 18 +18x = 40x

Изолировать переменную

22x = 18

x = 18/22

2 Упростить

x = 9/11

 

0972

LCD = 8

Умножить каждую дробь на LCD,

=> 4 +4x = 1 +2x

Изолировать x;

2x = -3

x = -1,5

Решение уравнений

Горячая математика

Решение уравнений с одной переменной

Ан уравнение это математическое утверждение, образованное путем помещения знака равенства между двумя числовыми или переменными выражениями, как в 3 Икс + 5 знак равно 11 .

А решение к уравнению это число который можно подключить для переменная чтобы сделать истинное утверждение числа.

Пример 1:

Замена 2 за Икс в

3 Икс + 5 знак равно 11

дает

3 ( 2 ) + 5 знак равно 11 , что говорит 6 + 5 знак равно 11 ; это правда!

Так 2 является решением.

Фактически, 2 является ЕДИНСТВЕННЫМ решением 3 Икс + 5 знак равно 11 .

Некоторые уравнения могут иметь более одного решения, бесконечно много решений или вообще не иметь решений.

Пример 2:

Уравнение

Икс 2 знак равно Икс

имеет два решения, 0 а также 1 , поскольку

0 2 знак равно 0 а также 1 2 знак равно 1 . Ни один другой номер не работает.

Пример 3:

Уравнение

Икс + 1 знак равно 1 + Икс

верно для все действительные числа . Оно имеет бесконечно много решения.

Пример 4:

Уравнение

Икс + 1 знак равно Икс

является никогда верно для Любые настоящий номер. Оно имеет нет решений .

установлен содержащий все решения уравнения, называется набор решений для этого уравнения.

Уравнение

Набор решений

3 Икс + 5 знак равно 11

{ 2 }

Икс 2 знак равно Икс

{ 0 , 1 }

Икс + 1 знак равно 1 + Икс

р (набор всех действительных чисел)

Икс + 1 знак равно Икс

(пустой набор)

Иногда вас могут попросить решить уравнение относительно определенного домен . Здесь возможности для значений Икс ограничены.

Пример 5:

Решите уравнение

Икс 2 знак равно Икс

через домен { 0 , 1 , 2 , 3 } .

Это немного сложное уравнение; это не линейный и это не квадратичный , поэтому у нас нет хорошего метода для ее решения. Однако, поскольку домен содержит только четыре числа, мы можем просто использовать метод проб и ошибок.

0 2 знак равно 0 знак равно 0 1 2 знак равно 1 знак равно 1 2 2 ≠ 2 3 2 ≠ 3

Итак набор решений над данным доменом { 0 , 1 } .

Решение уравнений с двумя переменными

Решениями уравнения с одной переменной являются числа . С другой стороны, решения уравнения с двумя переменными есть упорядоченные пары в виде ( а , б ) .

Пример 6:

Уравнение

Икс знак равно у + 1

верно, когда Икс знак равно 3 а также у знак равно 2 . Итак, заказанная пара

( 3 , 2 )

является решением уравнения.

Есть бесконечно много других решений этого уравнения, например:

( 4 , 3 ) , ( 11 , 10 ) , ( 5,5 , 4,5 ) , и т. п.

Упорядоченные пары, являющиеся решениями уравнения с двумя переменными, можно изобразить на графике. декартова плоскость . Результатом может быть линия или интересная кривая, в зависимости от уравнения. Смотрите также построение линейных уравнений а также построение квадратных уравнений .

Решение уравнения. Методы, приемы и примеры

Решение уравнения включает в себя нахождение значений неизвестных переменных в заданном уравнении. Условие равенства двух выражений удовлетворяется значением переменной. Решение линейного уравнения с одной переменной дает единственное решение, решение линейного уравнения с двумя переменными дает два результата. Решение квадратного уравнения дает два корня. Существует множество методов и процедур, применяемых при решении уравнения. Давайте подробно обсудим методы решения уравнения по одному.

1. В чем смысл решения уравнений?
2. шагов решения уравнения
3. Решение уравнений с одной переменной
4. Решение квадратного уравнения
5. Решение рационального уравнения
6. Решение радикального уравнения
7. Часто задаваемые вопросы о решении уравнений

В чем смысл решения уравнений?

Решение уравнений вычисляет значение неизвестной переменной, все еще уравновешивая уравнение с обеих сторон. Уравнение — это условие для переменной, при котором два выражения в переменной имеют одинаковое значение. Значение переменной, для которой выполняется уравнение, называется решением уравнения. Уравнение остается тем же, если поменять местами левую и правую части. Выделяется переменная, для которой нужно найти значение, и получается решение. Решение уравнения зависит от того, с каким типом уравнения мы имеем дело. Уравнения могут быть линейными уравнениями, квадратными уравнениями, рациональными уравнениями или радикальными уравнениями.

шагов решения уравнения

Цель решения уравнения состоит в том, чтобы найти значение переменной, удовлетворяющее условию истинности уравнения. Чтобы изолировать переменную, выполняются следующие операции, все еще уравновешивающие уравнение с обеих сторон. Таким образом, левая сторона остается равной правой, и, в конце концов, баланс не нарушается.

  • Добавление свойства равенства: Прибавьте одинаковое число к обеим сторонам. Если a = b, то a + c = b + c
  • Свойство равенства вычитания: вычитание одинакового числа с обеих сторон. Если а = b, то а — с = b — с
  • Свойство равенства умножения: умножить одно и то же число с обеих сторон. Если a = b, то ac = bc
  • Свойство равенства деления: Делим на одно и то же число в обе стороны. Если a = b, то a/c = b/c (где c ≠ 0)

После выполнения этого систематического уравновешивающего метода решения уравнения с помощью серии идентичных арифметических операций с обеих сторон уравнения мы разделяем переменную на одной из сторон, и последним шагом является решение уравнения.

Решение уравнений с одной переменной

Линейное уравнение одной переменной имеет вид ax + b = 0, где a, b, c — действительные числа. При решении линейного уравнения выполняются следующие шаги.

  • Удалите скобки и при необходимости используйте свойство распределения.
  • Упростите обе части уравнения, объединив одинаковые члены.
  • Если есть дроби, умножьте обе части уравнения на наименьший общий знаменатель всех дробей.
  • Если есть десятичные дроби, умножьте обе части уравнения на меньшую степень 10, чтобы преобразовать их в целые числа.
  • Перенесите переменные члены в одну часть уравнения, а постоянные члены в другую, используя свойства равенства сложения и вычитания.
  • Сделать коэффициент переменной равным 1, используя свойства умножения или деления на равенство.
  • изолировать переменную и получить решение.

Рассмотрим следующий пример: 3(x + 4) = 24 + x

Мы упрощаем LHS, используя свойство дистрибутивности.

3x + 12 = 24 + x

Сгруппируйте одинаковые термины вместе, используя метод транспонирования. Это становится 3x — x = 24-12

. Упрощаем дальше ⇒ 2x = 12

. Используйте свойство равенства деления, 2x/2 = 12/2

, изолируем переменную x. x = 6 является решением уравнения.

Используйте любой из следующих методов, чтобы упростить линейное уравнение и найти неизвестную переменную. Метод проб и ошибок, метод балансировки и метод транспонирования используются для выделения переменной.

Решение уравнения методом проб и ошибок

Предположим, что 12x = 60. Чтобы найти x, мы интуитивно пытаемся найти, что число, умноженное на 12, равно 60. Мы находим, что 5 — это искомое число. Решить уравнения методом проб и ошибок не всегда просто.

Решение уравнения методом балансировки

Нам нужно изолировать переменную x для решения уравнения. Для ее решения воспользуемся методом разделения переменных или методом балансировки. Рассмотрим уравнение 2x + 3 = 17,9. 0003

Сначала мы исключаем 3 на первом шаге. Чтобы сохранить баланс при решении уравнения, мы вычитаем 3 из каждой части уравнения.

Таким образом, 2x + 3 — 3 = 17 — 3

У нас есть 2x = 14

Теперь, чтобы изолировать x, мы делим на 2 с обеих сторон. (Свойство равенства деления)

2x/2 = 14/2

x = 7

Таким образом, мы изолируем переменную, используя свойства равенства при решении уравнения в методе уравновешивания.

Решение уравнения методом транспонирования

Решая уравнение, мы меняем стороны чисел. Этот процесс называется транспонированием. При перестановке числа мы меняем его знак или выполняем обратную операцию. Рассмотрим 5y + 2 = 22.

Нам нужно найти y, поэтому изолируем его. Следовательно, мы переносим число 2 на другую сторону. Уравнение принимает следующий вид:

5y = 22-2

5y = 20

Теперь, переставив 5 на другую сторону, мы обратим операцию умножения на деление. у = 20/5 = 4

Решение квадратного уравнения

Существуют уравнения, которые дают более одного решения. Квадратные многочлены имеют степень два, а нули квадратного многочлена представляют собой квадратное уравнение.

Рассмотрим (x+3) (x+2)= 0. Это квадратично по своей природе. Мы просто приравниваем каждое из выражений в LHS к 0.

Либо x+3 = 0, либо x+2 =0.

Мы получаем x = -3 и x = -2.

Квадратное уравнение имеет вид ax 2 + bx + c = 0. Решение квадратного уравнения дает два корня: α и β.

Шаги, необходимые для решения квадратного уравнения:

  • Путем выполнения метода квадратов
  • По методу факторизации
  • Методом формулы

Путем выполнения метода квадратов

Решить уравнение квадратного типа путем выполнения метода квадратов довольно просто, если применить наши знания об алгебраическом тождестве: (a+b) 2

  • Запишите уравнение в стандартной форме ax 2 + bx + c = 0,
  • Разделите обе части уравнения на a.
  • Переместить постоянный член на другую сторону
  • Добавьте квадрат половины коэффициента x с обеих сторон.
  • Дополните левую часть квадратом и упростите правую часть.
  • Извлеките квадратный корень из обеих сторон и найдите x.

Для получения дополнительной информации о решении уравнений (квадратичных) путем заполнения квадратов, нажмите здесь.

Методом факторизации

Решая уравнение квадратного типа методом факторизации, выполните шаги, описанные здесь. Запишите данное уравнение в стандартной форме и, разделив средние члены, разложите уравнение на множители. Перепишите полученное уравнение как произведение двух линейных множителей. Приравняйте каждый линейный множитель к нулю и найдите x. Рассмотрим 2x 2 + 19х + 30 = 0. Это стандартная форма ax 2 + bx + c = 0.

Разделите средний член таким образом, чтобы произведение членов было равно произведению коэффициента x 2 и c и суммы из терминов должно быть b. Здесь произведение слагаемых должно быть 60, а сумма должна быть 19. Таким образом, разделите 19x на 4x и 15x (поскольку сумма 4 и 15 равна 19, а их произведение равно 60).

2x 2 + 4x + 15x + 30 = 0

Вычтите общий делитель из первых двух членов и общие делители из двух последних членов.

2х(х+2)+15(х+2)=0 2 и x = -15/2

Решение квадратного уравнения включает такие шаги при разделении средних членов при факторизации.

Формульным методом

Решение уравнения квадратного типа по формуле

x = [-b ± √[(b 2 -4ac)]/2a помогает найти корни квадратного уравнения ax 2 + bx + c = 0. Подставляя значения a, b и c в формулу, мы приходим к решению.

Рассмотрим пример: 9x 2 -12 x + 4 = 0

a = 9, b = -12 и c = 4

x = [-b ± √[(b 2 -4ac)] /2a

= [12 ± √[((-12) 2 -4×9×4)] / (2 × 9)

= [12 ± √(144 — 144)] / 18

= (12 ± 0)/18

х = 12/18 = 2/3

Решение рационального уравнения

Уравнение, в знаменателе которого есть хотя бы одно полиномиальное выражение, называется рациональным уравнением. Решение рационального уравнения включает следующие шаги. Приведите дроби к общему знаменателю, а затем решите уравнение числителей.

Рассмотрим x/(x-1) = 5/3

При перекрестном умножении получаем

3x = 5(x-1)

3x = 5x — 5

3x — 5x = — 5

-2x = -5

x = 5/2

Решение радикального уравнения

Уравнение, в котором переменная находится под радикалом, называется радикальным уравнением. Решение уравнения, которое является радикалом, включает несколько шагов. Выразите данное радикальное уравнение через индекс радикала и уравновесьте уравнение. Решите для переменной.

Рассмотрим √(x+1) = 4

Теперь возведите обе стороны в квадрат, чтобы сбалансировать. [ √(x+1)] 2 = 4 2

(x+1) = 16

Таким образом, x = 16-1 =15

Важные замечания по решению уравнений: 0

    3 уравнение находит значение переменной в уравнении.
  • Решение уравнения удовлетворяет условию данного уравнения.
  • Решить уравнение линейного типа можно и графически.
  • Если правая часть уравнения равна нулю, то для решения уравнения просто начертите на графике левую часть уравнения, и точка пересечения x на графике будет решением(ями).

Статьи по теме:

  • Калькулятор решений уравнений
  • Синхронные линейные уравнения
  • Линейные уравнения и неравенства с одной переменной
  • Простые уравнения и их приложения

Часто задаваемые вопросы о решении уравнений

Что такое решение уравнения?

Решение уравнения — это нахождение значений неизвестных переменных в данном уравнении. Процесс решения уравнения зависит от типа уравнения.

Какие этапы решения уравнений?

Определите тип уравнения: линейное, квадратичное, логарифмическое, показательное, радикальное или рациональное.

  • Удалите скобки, если они есть в данном уравнении. Примените распределительное свойство.
  • Добавьте одинаковое число с обеих сторон
  • Вычесть одинаковое число с обеих сторон
  • Умножить одинаковое число с обеих сторон
  • Разделить на одно и то же число в обе стороны.

Золотое правило решения уравнения?

Идентифицирован тип уравнения. Если это линейное уравнение, используется метод разделения переменных или метод транспонирования. Если это квадратное уравнение, то используется достраивание квадратов, разбиение средних членов с помощью факторизации или по формульному методу.

Как вы используете 3 шага в решении уравнения?

3 шага решения уравнения:

  • удалить скобки, если они есть, используя свойство распределения,
  • упростить уравнение, добавляя или вычитая одинаковые члены,
  • выделение переменной и ее решение.

Как вы решаете линейные уравнения?

Решая линейное уравнение, мы изолируем переменную, значение которой нужно найти. Мы либо используем метод транспонирования, либо метод балансировки.

Как решать квадратные уравнения?

Решая квадратное уравнение, мы записываем уравнение в стандартной форме ax 2 + bx + c = 0, а затем решаем, используя метод формул или метод факторизации или завершая метод квадратов.

Как решать радикальные уравнения?

При решении радикального уравнения убираем знак подкореня, возводя обе части уравнения в индекс подкореня, изолируем переменную и находим x.

Как решать рациональные уравнения?

Решая рациональное уравнение, мы упрощаем выражение в каждой части уравнения, умножаем перекрестно, комбинируем одинаковые члены и затем изолируем переменную, чтобы найти x.

4.2 Решение линейных уравнений | Уравнения и неравенства

Предыдущий

4.1 Введение

Следующий

4. 3 Решение квадратных уравнений

4.2 Решение линейных уравнений (EMA34)

Самое простое уравнение для решения — это линейное уравнение. Линейное уравнение – это уравнение, в котором наибольшее показатель степени переменной равен \(\text{1}\). Ниже приведены примеры линейных уравнений:

\начать{выравнивать*} 2х+2&=1\ \frac{2 — x}{3x + 1} & = 2 \\ 4\влево(2x — 9\вправо) — 4x & = 4 — 6x \\ \frac{2a — 3}{3} — 3a & = \frac{a}{3} \конец{выравнивание*}

Решение уравнения означает нахождение значения переменной, которая делает уравнение верным. Например, чтобы решить простое уравнение \(x + 1 = 1\), нам нужно определить значение \(x\), которое сделает левый ручная сторона равна правой. Решение \(x = 0\).

Решение, также называемое корнем уравнения, представляет собой значение переменной, удовлетворяющей уравнению. Для линейных уравнений существует не более одного решения уравнения.

Для решения уравнений мы используем алгебраические методы, которые включают раскрытие выражений, группировку терминов и факторизация.

Например:

\начать{выравнивать*} 2х+2&=1\ 2x & =1 — 2 \quad \text{ (переставить)} \\ 2x & = -1 \quad \text{ (упростить)} \\ x & = -\frac{1}{2} \quad \text{(разделить обе части на } 2\text{)} \конец{выравнивание*}

Проверьте ответ, подставив \(x=-\frac{1}{2}\).

\начать{выравнивать*} \text{LHS} & = 2x + 2 \\ & = 2\влево(-\фракция{1}{2}\вправо) + 2 \\ &=-1+2\ & = 1 \\ \text{правая сторона} & =1 \конец{выравнивание*}

Следовательно, \(x=-\frac{1}{2}\)

Следующее видео дает введение в решение линейных уравнений.

Видео: 2F9B

Метод решения линейных уравнений (ЕМА35)

Общие шаги решения линейных уравнений:

  1. Раскройте все скобки.

  2. Переставьте члены так, чтобы все члены, содержащие переменную, находились на одной стороне уравнения, а все постоянные члены находятся на другой стороне.

  3. Сгруппируйте похожие термины вместе и упростите.

  4. Факторизация при необходимости.

  5. Найдите решение и запишите ответ.

  6. Проверьте ответ, подставив решение обратно в исходное уравнение.

Уравнение всегда должно быть сбалансировано, что бы вы ни делали с левой частью, вы должны делать и с левой справа.

Рабочий пример 1: Решение линейных уравнений

Найдите \(x\):

\[4(2x — 9) — 4x = 4 — 6x\]

Раскройте скобки и упростите

\начать{выравнивать*} 4(2х — 9) — 4х & = 4 — 6х \\ 8х — 36 — 4х & = 4 — 6х\ 8х — 4х + 6х & = 4 + 36\ 10x & = 40 \end{выравнивание*}

Разделить обе стороны на 10

\[х = 4\]

Проверьте ответ, подставив решение обратно в исходное уравнение

\начать{выравнивать*} \text{LHS} & = 4[2(4) — 9] — 4(4) \\ & = 4(8 — 9) — 16 \\ & = 4(-1) — 16 \\ &=-4 — 16\ &=-20\\ \text{RHS} & = 4 — 6(4) \\ &=4 — 24\ &=-20\\ \поэтому \text{левый } = \text{правый} \конец{выравнивание*}

Поскольку обе стороны равны, ответ правильный.

температура текст

Рабочий пример 2: Решение линейных уравнений

Найдите \(x\):

\[\frac{2 — x}{3x + 1} = 2\]

Умножить обе части уравнения на \(\left(3x + 1\right)\)

Деление на \(\text{0}\) не определено, поэтому должно быть ограничение: \(\left(x\) пе -\frac{1}{3}\right)\).

\начать{выравнивать*} \frac{2 — x}{3x + 1} & = 2 \\ (2 — х) & = 2(3х + 1) \конец{выравнивание*}

Раскройте скобки и упростите

\начать{выравнивать*} 2 — х&=6х+2\ -х — 6х & = 2 — 2\ -7x & = 0 \конец{выравнивание*}

Разделить обе стороны на \(-\text{7}\)

\начать{выравнивать*} х & = \ гидроразрыва {0}{-7} \\ х & = 0 \конец{выравнивание*}

Проверьте ответ, подставив решение обратно в исходное уравнение

\начать{выравнивать*} \text{LHS} & = \frac{2 — (0)}{3(0) + 1} \\ & = 2 \\ & = \text{Правая} \конец{выравнивание*}

Поскольку обе стороны равны, ответ правильный.

температура текст

Рабочий пример 3: Решение линейных уравнений

Решите для \(а\): \[\frac{2a — 3}{3} — 3a = \frac{a}{3}\]

Умножить уравнение на общий знаменатель \(\text{3}\) и упростить

\начать{выравнивать*} 2а — 3 — 9а & = а \\ -7а — 3 & = а \end{выравнивание*}

Переставить термины и упростить

\начать{выравнивать*} -7а — а&=3\ -8а & = 3 \конец{выравнивание*}

Разделить обе стороны на \(-\text{8}\)

\[a= -\frac{3}{8}\]

Проверьте ответ, подставив решение обратно в исходное уравнение

\начать{выравнивать*} \text{LHS} & = \frac{2\left(-\frac{3}{8}\right) — 3}{3} — 3\left(-\frac{3}{8}\right) \ \ & = \ гидроразрыва {\ влево (- \ гидроразрыва {3} {4} \ справа) — \ гидроразрыва {12} {4}} {3} + \ гидроразрыва {9{8} \\ & = \left[-\frac{15}{4}\times \frac{1}{3}\right] + \frac{9}{8} \\ & = -\frac{5}{4} + \frac{9}{8} \\ & = -\frac{10}{8} + \frac{9}{8} \\ & = -\frac{1}{8} \\ \text{RHS} & = \frac{-\frac{3}{8}}{3} \\ & = \frac{-\frac{3}{8}}{3} \\ & = -\frac{3}{8}\times \frac{1}{3} \\ & = -\frac{1}{8} \\ \поэтому \text{левый } = \text{правый} \конец{выравнивание*}

Поскольку обе стороны равны, ответ правильный.

температура текст

Учебник Упражнение 4.1

\(2y — 3 = 7\)

\begin{align*} 2у — 3&=7\ 2у&=10\ у & = 5 \end{выравнивание*}

\(2c = c-8\)

\begin{выравнивание*} 2с &= с — 8\\ с & = -8 \end{выравнивание*}

\(3 = 1 — 2c\)

\begin{align*} 3 &= 1 — 2с\\ 2с &= 1 — (3)\\ 2с & = -2\\ с & = \ гидроразрыва {-2} {2} \\ & = -1 \end{выравнивание*}

\(4b+5 = -7\)

\begin{выравнивание*} 4б +5 &= -7\\ 4b &= -7 — (5)\\ 4b&=-12\\ б & = \фракция{-12}{4}\\ & = -3 \end{выравнивание*}

\(-3y = 0\)

\begin{align*} -3у&=0\ у & = 0 \end{align*}

\(16y + 4 = -10\)

\begin{align*} 16у+4&=-10\ 16у&=-14\ y & = -\frac{14}{16}\\ & = -\фракция{7}{8} \end{выравнивание*}

\(12y + 0 = 144\)

\begin{выравнивание*} 12у + 0 & = 144\ 12у&=144\ у & = 12 \end{align*}

\(7 + 5y = 62\)

\begin{align*} 7+5у&=62\ 5у&=55\ у & = 11 \end{align*}

\(55 = 5x + \frac{3}{4}\)

\begin{align*} 55 & = 5x + \frac{3}{4} \\ 220&=20х+3\ 20х & = 217\ х & = \ гидроразрыва {217} {20} \end{выравнивание*}

\(5x = 2x + 45\)

\begin{align*} 5х&=2х+45\ 3х&=45\ х & = 15 \end{align*}

\(23x — 12 = 6 + 3x\)

\begin{align*} 23х — 12 и = 6 + 3х\ 20х & = 18\ х & = \ гидроразрыва {18} {20} \\ & = \фракция{9}{10} \end{выравнивание*}

\(12 — 6x + 34x = 2x — 24 — 64\)

\begin{align*} 12 — 6х + 34х & = 2х — 24 — 64\ 12+28х&=2х-88\ 26х&=-100\ х & = -\фракция{100}{26} \\ & = -\фракция{50}{13} \end{align*}

\(6x + 3x = 4 — 5(2x — 3)\)

\begin{align*} 6х + 3х & = 4 — 5 (2х — 3) \\ 9х & = 4 — 10х + 15\ 19х&=19\ х & = 1 \end{align*}

\(18 — 2p = p + 9\)

\begin{align*} 18 — 2п&=п+9\ 9&=3п\ р & = 3 \end{align*}

\(\dfrac{4}{p} = \dfrac{16}{24}\)

\begin{align*} \frac{4}{p} & = \frac{16}{24} \\ (4)(24) & = (16)(р) \\ 16р&=96\ р & = 6 \end{align*}

\(-(-16 — p) = 13p — 1\)

\begin{align*} -(-16 — п)&=13п — 1\ 16+п&=13п-1\ 17 и = 12п\ р & = \ гидроразрыва {17} {12} \end{align*}

\(3f — 10 = 10\)

\begin{align*} 3ф — 10 и = 10\ 3ф&=20\ f & = \frac{20}{3} \end{выравнивание*}

\(3f + 16 = 4f — 10\)

\begin{align*} 3ф + 16 & = 4ф — 10\ f & = 26 \end{align*}

\(10f + 5 = -2f -3f + 80\)

\begin{align*} 10ф+5&=-2ф-3ф+80\ 10f+5&=-5f+80\ 15ф&=75\ f & = 5 \end{выравнивание*}

\(8(f — 4) = 5(f — 4)\)

\begin{align*} 8(f — 4) & = 5(f — 4) \\ 8ф — 32 и = 5ф — 20\ 3ф&=12\ f & = 4 \end{align*}

\(6 = 6(f + 7) + 5f\)

\begin{align*} 6 & = 6(f + 7) + 5f \\ 6&=6ф+42+5ф\ -36&=11f\ f & = -\frac{36}{11} \end{выравнивание*}

\(-7x = 8(1 — x)\)

\begin{align*} -7х & = 8(1 — х) \\ -7х&=8 — 8х\ х & = 8 \end{align*}

\(5 — \dfrac{7}{b} = \dfrac{2(b + 4)}{b}\)

\begin{align*} 5 — \frac{7}{b} & = \frac{2(b + 4)}{b} \\ \frac{5b — 7}{b} & = \frac{2b + 8}{b} \\ 5б — 7 и = 2б + 8\ 3б и = 15\\ б & = 5 \end{выравнивание*}

\(\dfrac{x + 2}{4} — \dfrac{x — 6}{3} = \dfrac{1}{2}\)

\begin{align*} \frac{x + 2}{4} — \frac{x — 6}{3} & = \frac{1}{2} \\ \frac{3(x + 2) — 4(x — 6)}{12} & = \frac{1}{2} \\ \frac{3x + 6 — 4x + 24}{12} & = \frac{1}{2} \\ (-х + 30)(2) & = 12 \\ -2x + 60 & = 12\\ -2х&=-48\ х & = 24 \end{выравнивание*}

\(1 = \dfrac{3a — 4}{2a + 6}\)

Обратите внимание, что \(a \neq — -3\)

\начать{выравнивать*} 1 &= \ frac {3a — 4} {2a + 6} \\ 2а + 6 &= 3а — 4 \\ а &= 10 \конец{выравнивание*}

\(\dfrac{2-5a}{3} — 6 = \dfrac{4a}{3} +2 — a\)

\begin{align*} \frac{2-5a}{3} — 6 &= \frac{4a}{3} +2 — a \\ \frac{2-5a}{3} — \frac{4a}{3} + a &= 8 \\ \frac{2-5a — 4a + 3a}{3} &= 8 \\ 2 — 6а &= 24\ 6а&=-22\ а &= -\фракция{22}{6} \end{выравнивание*}

\(2 — \dfrac{4}{b+5} = \dfrac{3b}{b+5}\)

Примечание \(b \neq -5\)

\начать{выравнивать*} 2 — \frac{4}{b+5} &= \frac{3b}{b+5} \\ 2 &= \frac{3b+4}{b+5} \\ 2б + 10 &= 3б + 4 \\ б &= 6 \конец{выравнивание*}

\(3 — \dfrac{y — 2}{4} = 4\)

\begin{align*} 3 — \frac{y — 2}{4} & = 4 \\ -\frac{y — 2}{4} & = 1 \\ -у+2&=4\ у & = -2 \end{выравнивание*}

\(\text{1,5}x + \text{3,125} = \text{1,25}x\)

\begin{align*} \text{1,5}x + \text{3,125} & = \text{1,25}x \\ \text{1,5}x — \text{1,25}x & = -\text{3,125} \\ \text{0,25}x & = -\text{3,125} \\ х & = -\текст{12,5} \end{align*}

\(\text{1,3}(\text{2,7}x + 1) = \text{4,1} — x\)

\begin{align*} \text{1,3}(\text{2,7}x + 1) &= \text{4,1} — x \\ \text{3,51}x + \text{1,3} &= \text{4,1} — x \\ \text{4,51}x &= \text{2,8} \\ х & = \ гидроразрыва {\ текст {2,8}} {\ текст {4,51}} \\ &= \фракция{280}{451} \end{выравнивание*}

\(\text{6,5}x — \text{4,15}= 7 + \text{4,25}x\)

\begin{align*} \text{6,5}x — \text{4,15} &= 7 + \text{4,25}x \\ \text{2,25}x &= \text{11,15} \\ х & = \ гидроразрыва {\ текст {11,15}} {\ текст {2,25}} \\ & = \frac{\text{1 115}}{225} \\ &= \фракция{223}{45} \end{align*}

\(\frac{1}{3}P + \frac{1}{2}P — 10 = 0\)

\begin{выравнивание*} \frac{1}{3}P + \frac{1}{2}P — 10 & = 0 \\ \frac{2 + 3}{6}P & = 10 \\ 5П&=60\ Р & = 12 \end{align*}

\(1\frac{1}{4}(x — 1) — 1\frac{1}{2}(3x + 2) = 0\)

\begin{align* } 1\frac{1}{4}(x — 1) — 1\frac{1}{2}(3x + 2) & = 0 \\ \frac{5}{4}x — \frac{5}{4} — \frac{3}{2}(3x) — \frac{3}{2}(2) & = 0 \\ \frac{5}{4}x — \frac{5}{4} — \frac{9{2}x — \frac{6}{2} & = 0 \\ \frac{5 — 18}{4}x + \frac{-5 — 12}{4} & = 0 \\ \frac{-13}{4}x & = \frac{17}{4} \\ -13х&=17\ х & = -\фракция{17}{13} \end{align*}

\(\frac{1}{5}(x- 1) = \frac{1}{3}(x-2) + 3\)

\begin{align*} \frac{1}{5}(x- 1) &= \frac{1}{3}(x-2) + 3 \\ \frac{1}{5}x- \frac{1}{5} &= \frac{1}{3}x- \frac{2}{3} + 3 \\ -\frac{1}{5} + \frac{2}{3} — 3 &= \frac{2}{15}x \\ -\frac{38}{15} &= \frac{2}{15}x \\ х &= -\фракция{38}{2} \\ х &= -19\end{align*}

\(\dfrac{5}{2a} + \dfrac{1}{6a} — \dfrac{3}{a} = 2\)

\begin{align*} \frac{5}{2a} + \frac{1}{6a} — \frac{3}{a} & = 2 \\ \frac{5(3) + 1 — 3(6)}{6a} & = 2 \\ \frac{15 + 1 — 18}{6a} & = 2 \\ \frac{-2}{6a} & = 2 \\ -2&=12а\\ а & = -\фракция{1}{6} \end{выравнивание*}

Предыдущий

4. 1 Введение

Оглавление

Следующий

4.3 Решение квадратных уравнений

Решение линейных уравнений

1.7 Решение линейных уравнений

Цели обучения

  1. Использовать свойства равенства для решения основных линейных уравнений.
  2. Определите и решите условные линейные уравнения, тождества и противоречия.
  3. Удаление дробей из уравнений.
  4. Настройка и решение линейных приложений.

Решение основных линейных уравнений

Утверждение уравнения, указывающее, что два алгебраических выражения равны. это утверждение, указывающее, что два алгебраических выражения равны. Линейное уравнение с одной переменнойУравнение, которое можно записать в стандартной форме: ax+b=0, где a и b — действительные числа, а a≠0. , x — уравнение, которое можно записать в стандартной форме ax+b=0, где a и b — действительные числа, а a≠ 0. Например,

3x−12=0

РешениеЛюбое значение, которое может заменить переменную в уравнении для получения истинного утверждения. к линейному уравнению — это любое значение, которое может заменить переменную для получения истинного утверждения. Переменная в линейном уравнении 3x−12=0 равна x , а решение равно x=4. Чтобы убедиться в этом, подставьте значение 4 вместо 9.0140 x и проверьте правильность утверждения.

3x−12=03(4)−12=012−12=00=0    ✓

В качестве альтернативы, когда уравнение равно константе, мы можем проверить решение, подставив значение в вместо переменной и показав, что результат равен этой константе. В этом смысле мы говорим, что решения «удовлетворяют уравнению».

Пример 1

Является ли a=−12 решением уравнения −10a+5=25?

Решение:

Напомним, что при вычислении выражений рекомендуется сначала заменять все переменные скобками, а затем подставлять соответствующие значения. Используя круглые скобки, мы избегаем некоторых распространенных ошибок при работе с порядком операций.

−10a+5=−10(−12)+5=5+5=10≠25    ✗

Ответ: Нет, a=−12 не удовлетворяет уравнению.

Разработка методов решения различных алгебраических уравнений является одной из наших основных целей в алгебре. В этом разделе рассматриваются основные методы, используемые для решения линейных уравнений с одной переменной. Начнем с определения эквивалентных уравнений с одним и тем же набором решений. как уравнения с одним и тем же набором решений.

3x−5=16        3x=21        x=7}       эквивалентные уравнения

Здесь мы видим, что три линейных уравнения эквивалентны, потому что они имеют один и тот же набор решений, а именно {7}. Чтобы получить эквивалентные уравнения, используйте следующие свойства равенства Свойства, которые позволяют нам получать эквивалентные уравнения путем сложения, вычитания, умножения и деления обеих частей уравнения на ненулевые действительные числа. . Даны алгебраические выражения A и B , где c — ненулевое число:

Примечание: Умножение или деление обеих частей уравнения на 0 тщательно избегается. Деление на 0 не определено, а умножение обеих частей на 0 приводит к уравнению 0 = 0,

Мы решаем алгебраические уравнения, выделяя переменную с коэффициентом 1. Если дано линейное уравнение вида ax+b=c, то мы можем решить его в два этапа. Во-первых, используйте соответствующее свойство равенства сложения или вычитания, чтобы изолировать переменный термин. Затем изолируйте переменную, используя свойство равенства умножения или деления. Проверка решения в следующих примерах предоставляется читателю.

Пример 2

Решите: 7x−2=19.

Решение:

7x−2=197x−2 + 2=19 + 2         Добавить 2 к обеим  сторонам. 7x=217×7=217              Разделить обе части на 7.x=3

Ответ: Решение: 3.

Решение:

Если перед термином не стоит знак, он считается положительным. Другими словами, подумайте об этом как 56 = + 8 + 12 лет. Поэтому начнем с вычитания 8 по обе стороны от знака равенства.

56−8=8+12y−848=12y4812=12y124=y

Неважно, на какой стороне мы изолируем переменную, потому что свойство симметричности позволяет найти переменную по обе стороны от знака равенства, потому что x=5 эквивалентно 5=x. утверждает, что 4=y эквивалентно y=4.

Ответ: Решение: 4.

Пример 4

Решите: 53x+2=−8.

Решение:

Выделите переменный член, используя свойство сложения равенства, а затем умножьте обе части уравнения на величину, обратную коэффициенту 53.

53x+2=-853x+2-2=-8-2 Вычтите 2 на обе стороны.53x=−1035⋅53x=35⋅(−10)−2           Умножьте обе стороны на 35,1x=3⋅(−2)x=−6

3: 90 решение.

Таким образом, чтобы сохранить эквивалентные уравнения, мы должны выполнить одну и ту же операцию с обеих сторон уравнения.

Попробуйте! Решите: 23x+12=-56.

Ответ: x=−2

(нажмите, чтобы посмотреть видео)

Общие рекомендации по решению линейных уравнений

Обычно линейные уравнения не задаются в стандартной форме, поэтому их решение требует дополнительных шагов. При решении линейных уравнений цель состоит в том, чтобы определить, какое значение, если таковое имеется, даст истинное утверждение при подстановке в исходное уравнение. Для этого изолируйте переменную, выполнив следующие шаги:

  • Шаг 1: Упростите обе части уравнения, используя порядок операций, и объедините все одинаковые члены по одну сторону от знака равенства.
  • Шаг 2: Используйте соответствующие свойства равенства, чтобы объединить одинаковые термины на противоположных сторонах знака равенства. Цель состоит в том, чтобы получить переменный член в одной части уравнения и постоянный член в другой.
  • Шаг 3: Разделите или умножьте, если необходимо, чтобы изолировать переменную.
  • Шаг 4: Проверьте, решает ли ответ исходное уравнение.

Мы часто сталкиваемся с линейными уравнениями, в которых выражения по обе стороны от знака равенства могут быть упрощены. Если это так, то лучше сначала упростить каждую сторону, прежде чем решать. Обычно это включает в себя объединение односторонних терминов.

Примечание: На этом этапе нашего изучения алгебры использование свойств равенства должно показаться обычным. Поэтому отображение этих шагов в этом тексте, обычно синим цветом, становится необязательным.

Пример 5

Решите: −4a+2−a=1.

Решение:

Сначала объедините одинаковые члены слева от знака равенства.

−4a+2–a = 1 объедините сроки, похожие на то же самое.

Всегда используйте исходное уравнение для проверки правильности решения.

−4a+2−a=−4(15)+2−15=−45+21⋅55−15=−4+10+15=55=1      ✓

Ответ: решение 15.

Имея линейное уравнение в форме ax+b=cx+d, мы начинаем процесс решения, объединяя одинаковые члены по разные стороны от знака равенства. Для этого используйте свойство равенства сложения или вычитания, чтобы поместить одинаковые термины на одну сторону, чтобы их можно было комбинировать. В оставшихся примерах проверка предоставляется читателю.

Пример 6

Решите: −2y−3=5y+11.

Решение:

Вычтите 5y с обеих сторон, чтобы мы могли объединить члены, включающие и с левой стороны.

−2y−3−5y=5y+11−5y−7y−3=11

Отсюда решите, используя методы, разработанные ранее.

−7y — 3 = 11 Добавить 3 к обеим сторонам. — 7y = 14y = 14–7 Разделите обе стороны на −7.y = -2

Ответ: Решение составляет -2.

Решение часто требует применения распределительного свойства.

Пример 7

Решите: −12(10x−2)+3=7(1−2x).

Решение:

Сначала упростите линейные выражения по обе стороны от знака равенства.

−12 (10x -2)+3 = 7 (1-2x) Распределение. –5x+1+3 = 7–14x Объединение с такими же сторонами. Срок.9x = 3 Решания .x = 39 = 13

Ответ: Решение составляет 13.

Пример 8

Решение: 5 (3–a) -2 (5–2a) = 3.

Решение:

Начните с применения свойства распределения.

5(3−a)−2(5−2a)=315−5a−10+4a=35−a=3−a=−2

Здесь мы отмечаем, что −a эквивалентно −1a; поэтому мы решили разделить обе части уравнения на −1.

-a=-2-1a-1=-2-1a=2

В качестве альтернативы мы можем умножить обе части -a=-2 на минус единицу и получить тот же результат.

-a=-2(-1)(-a)=(-1)(-2)a=2

Ответ: решение 2.

Попробуйте! Решите: 6−3(4x−1)=4x−7.

Ответ: x=1

(нажмите, чтобы посмотреть видео)

Существуют три различных типа уравнений. До этого момента мы решали условные уравнения. Уравнения, которые верны для конкретных значений. Это уравнения, которые верны для конкретных значений. ТождествоУравнение, истинное для всех возможных значений. уравнение, верное для всех возможных значений переменной. Например, х = х      Идентичность имеет множество решений, состоящее из всех действительных чисел, ℝ. ПротиворечиеУравнение, которое никогда не бывает истинным и не имеет решения. уравнение, которое никогда не бывает истинным и, следовательно, не имеет решений. Например, х + 1 = х         Противоречие не имеет решения. Мы используем пустое множество Ø, чтобы показать, что решений нет.

Если конечным результатом решения уравнения является верное утверждение, например 0 = 0, то уравнение является тождеством, а любое действительное число является решением. Если решение приводит к ложному утверждению, например, 0 = 1, то уравнение является противоречием и решения нет.

Пример 9

Решите: 4(x+5)+6=2(2x+3).

Решение:

4(x+5)+6=2(2x+3)4x+20+6=4x+64x+26=4x+626=6   ✗

Решение приводит к ложному утверждению; следовательно, уравнение является противоречием и не имеет решения.

Ответ: Ø

Пример 10

Решите: 3(3y+5)+5=10(y+2)−y.

Решение:

3(3y+5)+5=10(y+2)−y9y+15+5=10y+20−y9y+20=9y+209y=9y0=0      ✓

Решение приводит к истинное утверждение; следовательно, уравнение является тождеством, а любое действительное число является решением.

Ответ: ℝ

Коэффициенты линейных уравнений могут быть любыми действительными числами, даже десятичными и дробными. В этом случае можно использовать свойство равенства умножения, чтобы очистить дробные коэффициенты и получить целые коэффициенты за один шаг. Если даны дробные коэффициенты, то умножьте обе части уравнения на наименьшее общее кратное знаменателей (НОК).

Пример 11

Решите: 13x+15=15x−1.

Решение:

Очистите дроби, умножив обе части на наименьшее общее кратное данных знаменателей. В данном случае это LCD(3,5)=15.

15om (13x+15) = 15om (15x — 1) Умножьте обе стороны на 15,15 ° С.13x+15om15 = 15om15x — 15-1 упростить.5x+3 = 3x — 15 Реша. 18x=−182=−9

Ответ: Решение равно −9.

Важно знать, что этот метод работает только для уравнений. Не пытайтесь очищать дроби при упрощении выражений. Напоминаем:

Упрощаем выражения и решаем уравнения. Если вы умножите выражение на 6, вы измените задачу. Однако, если вы умножите обе части уравнения на 6, вы получите эквивалентное уравнение.

Приложения, связанные с линейными уравнениями

Алгебра упрощает процесс решения реальных задач. Это делается путем использования букв для обозначения неизвестных, переформулирования задач в виде уравнений и предложения систематических методов решения этих уравнений. Чтобы решить задачи с помощью алгебры, сначала переведите формулировку задачи в математические утверждения, описывающие взаимосвязь между данной информацией и неизвестными. Обычно этот перевод в математические утверждения является трудным шагом в этом процессе. Ключом к переводу является внимательное прочтение задачи и выявление определенных ключевых слов и фраз.

При переводе предложений в математические выражения обязательно прочитайте предложение несколько раз и выделите ключевые слова и фразы. Важно сначала идентифицировать переменную, « пусть х представляет… » и описать словами, что такое неизвестная величина. Этот шаг не только делает нашу работу более читабельной, но и заставляет задуматься о том, что мы ищем.

Пример 12

Если из удвоенной суммы числа вычесть 6, получится 5. Найдите число.

Решение:

Пусть n представляет неизвестное число.

Чтобы понять, почему мы включили скобки в набор, вы должны изучить структуру следующих двух предложений и их переводы:

Ключевым моментом было сосредоточиться на фразе « удвоить сумму », это побудило нас сгруппируйте сумму в круглых скобках, а затем умножьте на 2. После перевода предложения в математическое выражение мы затем решаем.

2(n+8)−6=52n+16−6=52n+10=52n=−5n=−52

Проверить.

2(n+8)−6=2(−52+8)−6=2(112)−6=11−6=5             ✓

Ответ: число равно −52.

Ниже приведены общие рекомендации по составлению и решению текстовых задач.

  • Шаг 1: Прочитайте задачу несколько раз, определите ключевые слова и фразы и систематизируйте предоставленную информацию.
  • Шаг 2: Определите переменные, назначив букву или выражение неизвестным величинам.
  • Шаг 3: Переведите и составьте алгебраическое уравнение, моделирующее проблему.
  • Шаг 4: Решите полученное алгебраическое уравнение.
  • Шаг 5: Наконец, ответьте на вопрос в форме предложения и убедитесь, что он имеет смысл (проверьте).

А пока настройте все свои уравнения, используя только одну переменную. Избегайте двух переменных, ища взаимосвязь между неизвестными.

Пример 13

Периметр прямоугольника равен 92 метрам. Длина на 2 метра меньше ширины в 3 раза. Найдите размеры прямоугольника.

Решение:

Предложение « Длина на 2 метра меньше чем в 3 раза больше ширины » дает нам отношение между двумя переменными.

Пусть w представляет собой ширину прямоугольника.

Пусть 3w−2 обозначает длину.

Предложение « Прямоугольник имеет периметр измерения 92 метров ” предполагает алгебраическую схему. Подставьте 92 вместо периметра и выражение 3w−2 вместо длины в соответствующую формулу следующим образом: с одной переменной, решить для ширины, w .

92=2(3w−2)+2wРаспределить.92=6w−4+2wОбъединить подобные члены.92=8w−4Решить для w.96=8w12=w

Используйте 3w-2, чтобы найти длину.

l=3w−2=3(12)−2=36−2=34

Для проверки убедитесь, что периметр равен 92 метрам.

P=2l    +   2w=2(34)+2(12)=68+24=92

Ответ: Размеры прямоугольника 12 на 34 метра.

Пример 14

Учитывая годовую процентную ставку 438%, сколько времени потребуется 2500 долларов, чтобы получить 437,50 долларов простых процентов?

Решение:

Пусть t представляет собой время, необходимое для того, чтобы заработать 437,50 долларов при 438%. Организуйте информацию, необходимую для использования формулы простых процентов, I=prt.

Затем подставьте все известные величины в формулу и найдите единственную неизвестную, t .

I=prt437,50=2500(0,04375)t437,50=109,375t437,50109,375=109,375t109,3754=t

Ответ: Требуется 4 года на 2500 долларов, вложенных под 438% простых процентов, чтобы заработать 437,50 долларов.

Пример 15

Сьюзен вложила все свои сбережения в размере 12 500 долларов на два счета, приносящих простые проценты. Ее счет взаимных фондов заработал 7% в прошлом году, а ее компакт-диск заработал 4,5%. Если ее общая сумма процентов за год составила 670 долларов, сколько было на каждом счете?

Решение:

Связь между двумя неизвестными состоит в том, что они составляют 12 500 долларов. Когда речь идет об общей сумме, распространенный метод, используемый для того, чтобы избежать двух переменных, состоит в том, чтобы представить вторую неизвестную как разность общей суммы и первой неизвестной.

     Пусть x представляет собой сумму, инвестированную во взаимный фонд.

     Пусть 12 500 − 90 140 x 90 141 представляют оставшуюся сумму, вложенную в CD.

     Организуйте данные.

Общий процент представляет собой сумму процентов, полученных с каждого счета.

Проценты взаимного фонда+проценты CD = общий процент 0,07x+0,045 (12 500–x) = 670

Это уравнение моделирует проблему с одной переменной. Решите для x .

0,07x+0,045(12 500−x)  =6700,07x+562,5−0,045x=6700,025x+562,5=6700,025x=107,5x=107,50,025x=4,300

902,050

12 500−x=12 500−4 300=8 200

Ответ: Сьюзен вложила 4300 долларов под 7% во взаимный фонд и 8200 долларов под 4,5% в CD.

Ключевые выводы

  • Решение общих линейных уравнений включает выделение переменной с коэффициентом 1 по одну сторону от знака равенства. Для этого сначала используйте соответствующее свойство равенства сложения или вычитания, чтобы изолировать переменный член по одну сторону от знака равенства. Затем изолируйте переменную, используя свойство равенства умножения или деления. Наконец, убедитесь, что ваше решение решает исходное уравнение.
  • Если решение линейного уравнения приводит к верному утверждению, например, 0 = 0, то уравнение является тождеством, а набор решений состоит из всех действительных чисел, ℝ.
  • Если решение линейного уравнения приводит к ложному утверждению, например, 0 = 5, то уравнение является противоречием и решения нет, Ø.
  • Очистка дробей путем умножения обеих частей уравнения на наименьшее общее кратное всех знаменателей. Распределите и умножьте все члены на ЖК-дисплее, чтобы получить эквивалентное уравнение с целыми коэффициентами.
  • Упростите процесс решения реальных задач, создав математические модели, описывающие отношения между неизвестными. Используйте алгебру для решения полученных уравнений.

Тематические упражнения

    Часть A. Решение основных линейных уравнений

      Определить, является ли заданное значение решением.

    1. −5x+4=−1; х=−1

    2. 4x−3=−7; х=−1

    3. 3у-4=5; у=93

    4. -2у+7=12; у=-52

    5. 3а-6=18-а; а=-3

    6. 5(2т-1)=2-т; т=2

    7. ах-б=0; х=ба

    8. ось+b=2b; х=ба

      Решить.

    1. 5x−3=27

    2. 6x−7=47

    3. 4x+13=35

    4. 6x−9=18

    5. 9а+10=10

    6. 5−3a=5

    7. −8t+5=15

    8. −9t+12=33

    9. 23x+12=1

    10. 38x+54=32

    11. 1−3y5=2

    12. 2−5y6=−8

    13. 7−y=22

    14. 6−y=12

    15. Решите для x : ax-b=c

    16. Решить для x : ax+b=0

    Часть B: Решение линейных уравнений

      Решить.

    1. 6x−5+2x=19

    2. 7−2x+9=24

    3. 12x−2−9x=5x+8

    4. 16-3x-22=8-4x

    5. 5у-6-9у=3-2у+8

    6. 7−9 лет+12=3 года+11−11 лет

    7. 3+3а-11=5а-8-2а

    8. 2-3а=5а+7-8а

    9. 13x−32+52x=56x+14

    10. 58+15x−34=310x−14

    11. 1,2х-0,5-2,6х=2-2,4х

    12. 1,59−3,87x=3,48−4,1x−0,51

    13. 5−10x=2x+8−12x

    14. 8x−3−3x=5x−3

    15. 5(у+2)=3(2у−1)+10

    16. 7(у-3)=4(2у+1)-21

    17. 7−5(3t−9)=22

    18. 10−5(3t+7)=20

    19. 5−2x=4−2(x−4)

    20. 2(4x−5)+7x=5(3x−2)

    21. 4(4а-1)=5(а-3)+2(а-2)

    22. 6(2b-1)+24b=8(3b-1)

    23. 23(х+18)+2=13х−13

    24. 25x−12(6x−3)=43

    25. 1,2(2х+1)+0,6х=4х

    26. 6+0,5(7x−5)=2,5x+0,3

    27. 5(у+3)=15(у+1)−10у

    28. 3(4−у)−2(у+7)=−5у

    29. 15(2а+3)−12=13а+110

    30. 32а=34(1+2а)−15(а+5)

    31. 6−3(7x+1)=7(4−3x)

    32. 6(х-6)-3(2х-9)=-9

    33. 34(у-2)+23(2у+3)=3

    34. 54−12(4y−3)=25(y−1)

    35. −2(3x+1)−(x−3)=−7x+1

    36. 6(2x+1)−(10x+9)=0

    37. Решить для w : P=2l+2w

    38. Решите для a : P=a+b+c

    39. Решить для t : D=rt

    40. Решить для w : V=lwh

    41. Решить для b : A=12bh

    42. Решить для a : s=12at2

    43. Решите для a : A=12h(a+b)

    44. Решить для ч : V=13πr2ч

    45. Решите для F :  C=59(F−32)

    46. Решить для x : ax+b=c

    Часть C: Приложения

      Составьте алгебраическое уравнение и решите его.

      Проблемы с номером

    1. Если из суммы чисел вычесть 3 и 10, получится 2. Найдите число.

    2. Сумма умноженного на 3 числа и 12 равна 3. Найдите число.

    3. Трехкратная сумма числа и 6 равна пятикратному числу. Найдите число.

    4. Удвоенная сумма числа и 4 равна 3-кратной сумме числа и 1. Найдите число.

    5. Большее целое число в 1 раз больше другого целого числа более чем в 3 раза. Если сумма целых чисел равна 57, найдите целые числа.

    6. Большее целое число в 5 раз больше другого целого числа более чем в два раза. Если сумма целых чисел равна 83, найдите целые числа.

    7. Одно целое число в 3 раза меньше другого целого числа. Найдите целые числа, если их сумма равна 135.

    8. Одно целое число в 10 раз меньше другого целого числа более чем в 4 раза. Найдите целые числа, если их сумма равна 100.

    9. Сумма трех последовательных целых чисел равна 339. Найдите целые числа.

    10. Сумма четырех последовательных целых чисел равна 130. Найдите целые числа.

    11. Сумма трех последовательных четных целых чисел равна 174. Найдите целые числа.

    12. Сумма четырех последовательных четных целых чисел равна 116. Найдите целые числа.

    13. Сумма трех последовательных нечетных целых чисел равна 81. Найдите эти числа.

    14. Сумма четырех последовательных нечетных целых чисел равна 176. Найдите целые числа.

      Задачи по геометрии

    1. Длина прямоугольника на 5 см меньше его ширины в два раза. Найдите длину и ширину, если периметр равен 134 сантиметрам.

    2. Длина прямоугольника на 4 сантиметра больше, чем его ширина в 3 раза. Найдите длину и ширину, если периметр равен 64 см.

    3. Ширина прямоугольника равна половине его длины. Найдите размеры прямоугольника, если его периметр равен 36 см.

    4. Ширина прямоугольника на 4 дюйма меньше его длины. Найдите размеры прямоугольника, если его периметр равен 72 дюймам.

    5. Периметр квадрата равен 48 дюймам. Найдите длину каждой стороны.

    6. Периметр равностороннего треугольника равен 96 дюймам. Найдите длину каждой стороны.

    7. Длина окружности равна 80π единицам. Найдите радиус.

    8. Длина окружности 25 сантиметров. Найдите радиус, округлив его до сотых.

      Задачи на простые проценты

    1. На сколько лет нужно инвестировать 1000 долларов под 512%, чтобы заработать 165 долларов в виде простых процентов?

    2. На сколько лет нужно инвестировать 20 000 долларов под 614%, чтобы заработать 3 125 долларов в виде простых процентов?

    3. Под какую годовую процентную ставку нужно инвестировать 6500 долларов в течение 2 лет, чтобы получить 1040 долларов в виде простых процентов?

    4. Под какую годовую процентную ставку нужно инвестировать 5750 долларов в течение 1 года, чтобы получить 333,50 доллара в виде простых процентов?

    5. Если простые проценты, полученные за 5 лет, составили 1860 долларов, а годовая процентная ставка составила 6%, какова была основная сумма долга?

    6. Если простые проценты за 2 года составили 543,75 доллара, а годовая процентная ставка — 334%, какова была основная сумма долга?

    7. Сколько лет потребуется 600 долларов, чтобы удвоить простые проценты по ставке 5% годовых? (Подсказка: чтобы удвоить, инвестиции должны приносить 600 долларов в виде простых процентов. )

    8. Сколько лет потребуется 10 000 долларов, чтобы удвоить простые проценты по ставке 5% годовых? (Подсказка: чтобы удвоить, инвестиции должны приносить $10 000 в виде простых процентов.)

    9. Джим вложил 4200 долларов в два счета. Один счет зарабатывает 3% простых процентов, а другой зарабатывает 6%. Если проценты через 1 год составили 159 долларов, сколько он вложил в каждый счет?

    10. Джейн вложила свои сбережения в размере 6500 долларов на два счета. У нее есть часть его на компакт-диске под 5% годовых, а остальная часть на сберегательном счете, который приносит 4% годовых. Если простой процент, полученный с обоих счетов, составляет 303 доллара в год, то сколько у нее есть на каждом счете?

    11. Хосе положил прошлогодний бонус в размере 8400 долларов на два счета. Он вложил часть в компакт-диск с годовой процентной ставкой 2,5%, а остальную часть в фонд денежного рынка с годовой процентной ставкой 1,5%. Его общая сумма процентов за год составила 198 долларов. Сколько он вложил в каждый счет?

    12. Мэри вложила все свои сбережения в размере 3300 долларов на два счета. Ее счет взаимных фондов заработал 6,2% в прошлом году, а ее компакт-диск заработал 2,4%. Если ее общая сумма процентов за год составила 124,80 доллара, сколько было на каждом счете?

    13. Алиса вкладывает деньги на два счета, один с годовой процентной ставкой 3%, а другой с годовой процентной ставкой 5%. Она вкладывает в счет с более высокой доходностью в 3 раза больше, чем в счет с более низкой доходностью. Если ее общий процент за год составляет 126 долларов, сколько она вложила в каждый счет?

    14. Джеймс вложил наследство в два разных банка. Один банк предлагал 512% годовых, а другой 614%. Он вложил в более доходный банковский счет вдвое больше, чем в другой. Если его общая сумма простых процентов за 1 год составила 5760 долларов, то какова сумма его наследства?

      Задачи равномерного движения

    1. Если Джиму нужно 114 часов, чтобы проехать 40 миль до работы, то какова средняя скорость Джима?

    2. Джилл потратила 312 часов, чтобы проехать 189 миль домой из колледжа. Какова была ее средняя скорость?

    3. С какой скоростью должен ехать Джим, если он хочет проехать 176 миль за 234 часа?

    4. Джеймс и Мартин смогли проехать 1140 миль из Лос-Анджелеса в Сиэтл. Если общая поездка заняла 19 часов, то какова была их средняя скорость?

    Часть D: Дискуссионная доска

    1. Что считается основным делом алгебры? Объяснять.

    2. Каково происхождение слова алгебра ?

    3. Создайте свою собственную личность или противоречие и поделитесь ею на доске обсуждений. Предложите решение и объясните, как вы его нашли.

    4. Опубликуйте в этом разделе что-нибудь особенно полезное или интересное. Объяснить, почему.

    5. Выполните поиск в Интернете по запросу «решение линейных уравнений». Поделитесь ссылкой на веб-сайт или видеоурок, которые вы считаете полезными.

Ответы

  1. Да

  2. Да

  3. 6

  4. 112

  5. 0

  6. −54

  7. 34

  8. −3

  9. −15

  10. х=b+ca

  1. 3

  2. −5

  3. −172

  4. 78

  5. 2,5

  6. Ø

  7. 3

  8. 2

  9. Ø

  10. −53

  11. −81

  12. 1,2

  13. 0

  14. Ø

  15. 65

  16. w=P−2l2

  17. т=Др

  18. б=2Ач

  19. а=2Ач-б

  20. Ф=95С+32

  1. −5

  2. 9

  3. 14, 43

  4. 46, 89

  5. 112, 113, 114

  6. 56, 58, 60

  7. 25, 27, 29

  8. Ширина: 24 сантиметра; длина: 43 см

  9. Ширина: 6 дюймов; длина: 12 дюймов

  10. 12 дюймов

  11. 40 шт.

  12. 3 года

  13. 8%

  14. 6 200 долл. США

  15. 20 лет

  16. Он инвестировал 3100 долларов под 3% и 1100 долларов под 6%.

  17. Хосе вложил 7200 долларов в CD и 1200 долларов в фонд денежного рынка.

Добавить комментарий

Ваш адрес email не будет опубликован. Обязательные поля помечены *